Предел функции двух переменных: 1.2. Предел функции двух переменных. Непрерывность

Содержание

Предел и непрерывность функции двух переменных с примерами решения

Содержание:

Множества точек n-мерного евклидова пространства:

Назовем n-мерным координатным пространством и обозначим

Координатное пространство называется n-мерным евклидовым пространством и обозначается , если между двумя любыми точками пространства определено расстояние, обозначаемое и вычисляемое по формуле:

Опишем важнейшие типы множеств n-мерного евклидова пространства .

Множество всевозможных точек М пространства , координаты которых удовлетворяют:

  • а) неравенству
  • б) неравенству
  • в) равенству
  • г) неравенствам

где — некоторые положительные числа, называется:

  • а) открытым n-мерным шаром;
  • б) замкнутым n-мерным шаром;
  • в) n-мерной сферой радиуса R с центром в точке
  • г) открытым n-мерным координатным параллелепипедом с центром в точке

Открытый n-мерный шар раднуса с центром в точке будем называть -окрестностью точки

Множество {М} точек пространства называется ограниченным, если найдется n-мерный шар, содержащий все точки этого множества.

Определение 15.2.1. Пусть задано множество точек {М} n-мерного евклидова пространства . Если каждой точке М из множества {М) ставится в соответствие по известному закону некоторое число и, то говорят, что на множестве {м} задана функция u = u(х) или . При этом множество {M} называют областью задания функции . Число u соответствующее данной точке М из множества {M}, называется частным значением функции в точке М. Совокупность {м} всех частных значений функции называется множеством значений этой функции.

Частным случаем функции n переменных является функция двух переменных, которую можно определить следующим образом.

Если каждой паре (х, у) значений двух независимых друг от друга переменных величин х и у. принадлежащих области D, соответствует по известному закону определенное значение величины z, то называется функцией двух независимых переменных х и у, определенной в области D. D называется областью определения функции z = f(x, у).

Функция двух независимых переменных может быть задана:

  • аналитически, то есть с помощью формул;
  • при помощи таблицы, в которой указываются частные значения х и y, и соответствующие значения z;
  • при помощи компьютерной программы, в которой определен алгоритм получения значений функции z для частных значений х и y

Для определения частных значений функции двух переменных должны быть заданы значения независимых переменных: И поэтому ясно, что функция z =f(х, у) может быть определена на всей плоскости или в некоторой ее части, то есть в некоторой области. Область определения как всякое множество может быть открытой (незамкнутой) или замкнутой.

Рассмотрим функцию

определенную в области D на плоскости Оху и систему прямоугольных декартовых координат Oxyz. В каждой точке (x, у) восстановим перпендикуляр к плоскости Оху и на ней отложим отрезок, равный f(x,y). Тогда получим точку

Геометрическое место точек Р, координаты которых удовлетворяют (15.2.1), называется графиком функции двух переменных. Но уравнение (15.2.1) определяет некоторую поверхность, следовательно, графиком функции двух переменных является поверхность, проектирующаяся на плоскость Оху в область определения функции.

Например, областью задания функции двух переменных является круг радиуса 3 с центром в начале координат, а множество значений представляет собой отрезок.

Область задания функции n переменных служит n-мерный шар радиуса 2 с центром в точке O(0,0…..0). Множеством значений рассматриваемой функции является отрезок [0;2],

Из приведенных примеров, видим, что область задания функции n переменных представляет собой некоторое множество точек n-мерного евклидова пространства , а множество всех значений этой функции- множество одномерного евклидова пространства .

Предел функции двух переменных

Определение 15.3.1. (предел функции по Кохии) Число А называется пределом функции z = f(M) в точке (или при ). если для любого положительного числа найдется такое положительное число , что для любой точки М из множества задания этой функции, удовлетворяющей условию , справедливо неравенство .

Для обозначения предела функции в точке ис­пользуется символика: или если точка имеет координаты ,тo Отметим, что в определении не указывается закон стремления точки М к точке Л , поэтому он может быть любым. Функция называется бесконечно малой в точке Например, функция где — положительные числа, является бесконечно ма­лой в точке Если функция имеет предел А в точке , то ее можно представить в виде суммы предела и бесконечно малой функции в этой точке: где Для функции можно определить понятие по­вторного предела, то есть предела по одной из переменных при фиксированных значениях остальных переменных. Рассмотрим повторный предел для функции двух переменных Пусть функция задана в прямоугольной окрестности точки и пусть для каждого фиксированного удовлетворяющего условию существует предел функции одной пе­ ременной х в точке

Тогда, если существует предел А функции в точке то он называется для функции в точке , который обозначается следующим образом:

Аналогично определяется повторный предел: Отметим, что если в точке функция имеет пре­ дел равный и существуют пределы и то повторные пределы и существуют и оба равны А. Однако из существования повторных пределов еще не следует существования предела функции Так, например, функ­ция не имеет предела в точке хотя можно указать такие законы стремления точки , что повторные пределы будут равны: Но если точка М стремится к точке по прямой , то значение повторного предела будет зависеть от значения , что и свидетельствует об отсутствии предела рассматриваемой функции в точке (О,О).

Может оказаться, что оба повторных предела существуют, но различны. Нетрудно заметать, что для функции будем иметь различные повторные пределы:

Поэтому, можно сделать вывод, что если для любого закона движения точки М к точке повторные пределы существуют и они равны между собой, тогда и предел функции z = f(x, у) существует и равен этим повторным пределам.

Непрерывность функции двух переменных

Рассмотрим функцию п переменных , заданную на некотором множестве пространства . Пусть -произвольная точка , принадлежащая множеству и такая, что в любой -окрестности точки содержатся точки множества .

Будем говорить, что функция z = f(М) непрерывна в точке , если предел этой функции в точке существует и равен частному значению. Символически непрерывность функции z = f(М) в точке можно записать в виде:

В частности, для функции двух переменных:

что поскольку , то условие непрерывности можно записать в виде:

Значит, для непрерывной функции в точке символ lim предела и символ характеристики функции можно менять местами.

Точки пространства , в которых функция z = f(м) не обладает свойством непрерывности, называются точками разрыва этой функции.

Используя определение предела функции в точке по Коши, можно сформулировать определения непрерывности функции по Коши.

Если обозначить через полное приращение функции , определяемое по формуле , где М произвольная точка из области задания функции, то для непрерывности функции z = f(M) в точке необходимо и достаточно, чтобы ее приращение представляло в точке бесконечно малую функцию, то есть

Аналогичным образом определяется непрерывность функции по одной из переменных. Для этого вводятся частные приращения:

и тогда если , то функция называется непрерывной по переменной

Очевидно, что из условия непрерывности функции в данной точке вытекает непрерывность этой функции в точке по каждой из переменных . Однако, из непрерывности функции в точке по каждой из переменных не вытекает, вообще говоря, непрерывность функции в этой точке.

Для непрерывности функции n переменных справедливы многие аналогичные свойства непрерывных функций одной переменной: непрерывность сложной функции, устойчивость знака непрерывной функции, о прохождении через любое промежуточное значение, теоремы Вейрштраса и т.д.

Частные производные первого и высших порядков функции двух переменных

Пусть функция n переменных задана на множестве {м} и точка — внутренняя точка данного множества. Зафиксируем все аргументы, кроме , которому придадим произвольное приращение, такое, чтобы точка с координатами находилась в области задания функции. Определим соответствующее частное приращение функции в точке

. Составим отношение частного приращения к соответствующему приращению аргумента :

Отношение (15.5.1) представляет собой функцию , определенную для всех отличных от нуля значений , для которых точка принадлежит области задания функции

Определение 15.5.1.

Если существует предел отношения (15.5.1) частного приращения функции к соответствующему приращению аргумента , при стремлении к нулю, то тот предел называется частной производной функции в точке по аргументу и обозначается одним из символов:

Таким образом,

(15.5.2)

В частности, для функции двух переменных можно определить частные производные по х и у:

Из определения 15.5.1 следует, что частная производная функции по аргументу представляет собой обыкновенную производную функции одной переменной при фиксированных значениях остальных переменных. Поэтому вычисление частных производных производится с использованием правил вычислений производных функций одной переменной и таблицы производных.

  • Заказать решение задач по высшей математике

Пример:

Вычислить частные производные функций:

a)

Решение:

Для вычисления частных производных воспользуемся правилами вычисления производных функции одной переменной, при этом, вычисляя частную производную по одной из переменных, другую считаем постоянной величиной.

Предположим, что частная производная по аргументу функции , определенной на множестве , существует в каждой точке этого множества. Тогда указанная частная производная представляет собой функцию n переменных, определенную на множестве {М}. Если эта функция имеет частную производную по аргументу в некоторой точке множества {м), то она называется частной производной второго порядка функциив точке сначала по аргументу хк, а затем по аргументу х, и обозначается одним из символов: . При этом, если , то частная производная называется смешанной частной производной второго порядка. После того как нами введено понятие второй частной производной, последовательно вводится понятие третьей частной производной, четвертой и т.

д. Частную производную функции П0 одному из аргументов в некоторой точке от частной производной (n-l)-ro порядка по аргументам (отдельные или даже все номера которых могут совпадать) называется частной производной n-го порядка функции в точке .

В частности для функции двух переменных частные производные второго порядка определяются следующим образом.

Пусть частная производная по аргументу х функции z = f(х, у), определенной в области D. существует в каждой точке области D. В этом случае указанная частная производная представляет собой функцию двух переменных, также определенную в области D.

Тогда если эта функция имеет частную производную по аргументу х в некоторой точке , то указанную частную производную по х называют второй частной производной или частной производной второго порядка функции в точке М по аргументу х и обозначают одним из символов:

Аналогично определяется частная производная второго порядка по аргументу у.

Частная производная называется смешанной производной второго порядка, т. е.

После того как введено понятие второй частной производной, можно последовательно ввести понятие третьей частной производной, затем четвертой и т.д.

Так как частная производная функции по аргументу х (у) определяется как обыкновенная производная функции одной переменной х (у)при фиксированном значении другой переменной, то методика вычисления частных производных высших порядков предполагает умение вычислять только обыкновенные производные первого порядка.

Пример:

Вычислить частные производные второго порядка функции

Решение:

Применив правило вычисления частных производных, получим:

В рассмотренном примере смешанные частные производные равны друг другу. Вообще говоря, значения смешанных производных зависят от порядка, в котором производятся последовательные вычисления производных.

Достаточные условия независимости порядка вычисления производных определяются следующей теоремой.

Теорема 15. 5.1. Пусть функция в точке имеет непрерывные частные производные второго порядка, тогда в этой точке частные производные равны.

Более того имеет место такая же теорема о независимости значений любой смешанной частной производной n-го порядка от порядка, в котором производятся вычисления частных производных.

Теорема 15.5.2. Пусть функция имеет непрерывные частные производные n-го порядка в точке Тогда в этой точке значение любой смешанной частной производной n-го порядка не зависит от порядка, в котором производится вычисление производных.

Помогите найти предел функции двух переменных : Анализ-I

Сообщения без ответов | Активные темы | Избранное



Правила форума

В этом разделе нельзя создавать новые темы.

Если Вы хотите задать новый вопрос, то не дописывайте его в существующую тему, а создайте новую в корневом разделе «Помогите решить/разобраться (М)».

Если Вы зададите новый вопрос в существующей теме, то в случае нарушения оформления или других правил форума Ваше сообщение и все ответы на него могут быть удалены без предупреждения.

Не ищите на этом форуме халяву, правила запрещают участникам публиковать готовые решения стандартных учебных задач. Автор вопроса обязан привести свои попытки решения и указать конкретные затруднения.

Обязательно просмотрите тему Правила данного раздела, иначе Ваша тема может быть удалена или перемещена в Карантин, а Вы так и не узнаете, почему.


 
MaxMelnikov 

 Помогите найти предел функции двух переменных

28.03.2009, 18:17 

28/03/09
2

Никогда не решал такие примеры. Решал только с одной переменной, и те давались тяжело. Помогите найти предел или же доказать что его не существует:


   

                  

Brukvalub 

 

28.03.2009, 19:04 

Заслуженный участник

01/03/06
13626
Москва

Предела нет. Посмотрите, что будет в точках кривых

и


   

                  

MaxMelnikov 

 

29.03.2009, 00:58 

28/03/09
2

Т.е. найти пределы:

И если их не существует — значит и не существует и этого предела:

Можно поподробнее? С одной переменной просто намного легче


   

                  

Someone 

 

29. 03.2009, 01:11 

Заслуженный участник

23/07/05
17955
Москва

MaxMelnikov в сообщении #199797 писал(а):

Т.е. найти пределы:

Нет. Вам предлагалось подставить и в данную функцию и вычислить пределы двух получившихся функций при . Если исходный предел существует, то оба этих предела также будут существовать и будут иметь одинаковые значения (совпадающие с исходным пределом). Если же получатся разные пределы, то исходный предел не существует.


   

                  

ASA 

 

29. 03.2009, 19:15 

30/01/09
194

MaxMelnikov писал(а):


Такая запись вряд ли корректна. По-моему нужно так:

и речь, стало быть, идет о повторном интеграле, а не о двойном.


   

                  

Atata379 

 Re: Помогите найти предел функции двух переменных

09. 02.2012, 22:58 

09/02/12
4

Чтобы не создавать новую тему, спрошу в этой давнишней…
Я, как и автор темы тоже «Никогда не решал такие примеры. Решал только с одной переменной».

У меня такой вопрос: как получили , которые следует подставлять в исходный предел?

И как называется это правило/теорема:

Someone в сообщении #199801 писал(а):

Если исходный предел существует, то оба этих предела также будут существовать и будут иметь одинаковые значения (совпадающие с исходным пределом). Если же получатся разные пределы, то исходный предел не существует.

?


   

                  

Someone 

 Re: Помогите найти предел функции двух переменных

10.02.2012, 00:31 

Заслуженный участник

23/07/05
17955
Москва

Atata379 в сообщении #536851 писал(а):

У меня такой вопрос: как получили и , которые следует подставлять в исходный предел?

Догадались. А чтобы хорошо догадываться, надо иметь большой опыт в вычислении пределов.

Atata379 в сообщении #536851 писал(а):

И как называется это правило/теорема: …?

Как называется — не знаю, но формулируется так: если предел существует, то каждый его частичный предел тоже существует и имеет то же самое значение.

Что нужно понимать под частичным пределом, зависит от вида предела. Обычно это ограничение исходного предела на некоторое (не совсем произвольное) подмножество области определения функции, от которой вычисляется предел.


   

                  

Atata379 

 Re: Помогите найти предел функции двух переменных

10. 02.2012, 01:29 

09/02/12
4

Someone,

у этих есть какое-нибудь название? Судя по второму сообщению этой темы они называются «точки кривых»… хочу найти о них какую-нибудь информацию. Спасибо за ответ.


   

                  

Shtorm 

 Re: Помогите найти предел функции двух переменных

09. 07.2012, 22:46 

14/02/10
4956

MaxMelnikov в сообщении #199655 писал(а):

Никогда не решал такие примеры. Решал только с одной переменной, и те давались тяжело. Помогите найти предел или же доказать что его не существует:

Соответственно в продолжение темы:

А если нужно найти предел то нужно рассматривать на луче ? А какие тогда брать кривые? Тоже ? А можно тогда взять ??


   

                  

AKM 

 Re: Помогите найти предел функции двух переменных

10. 07.2012, 23:06 

Заблокирован по собственному желанию

18/05/09
3612

Достаточно взять , убедиться, что предел зависит от , т.е. предел зависит от пути, т.е. предела по двум переменным нет.
(Проверять другие возможные пути после этого нет надобности).
Возможно, Вам интереснее другой вариант — когда предел есть. Как тогда проверить все

возможные пути?
Вероятно, надо идти другим путём (в ленинском смысле этой фразы): не проверять какие-то семейства путей, а. ..


   

                  

Shtorm 

 Re: Помогите найти предел функции двух переменных

11.07.2012, 00:10 

14/02/10
4956

AKM в сообщении #594294 писал(а):

Достаточно взять , убедиться, что предел зависит от , т. е. предел зависит от пути, т.е. предела по двум переменным нет.
(Проверять другие возможные пути после этого нет надобности).

Да. Это единственное, что не вызывает сомнений в теме предела двух переменных.

AKM в сообщении #594294 писал(а):

Возможно, Вам интереснее другой вариант — когда предел есть. Как тогда проверить все возможные пути?
Вероятно, надо идти другим путём (в ленинском смысле этой фразы): не проверять какие-то семейства путей, а…

Посмотреть на график функции двух переменных? Или угадать предел, как кажется где-то ewert советовал?


   

                  

kw_artem 

 Re: Помогите найти предел функции двух переменных

11. 07.2012, 00:22 

17/01/12
445

Shtorm в сообщении #594312 писал(а):

Посмотреть на график функции двух переменных?

кажется в этом случае конечный предел на графике изобразится как асимптота-плоскость


   

                  

AKM 

 Re: Помогите найти предел функции двух переменных

11. 07.2012, 12:13 

Заблокирован по собственному желанию

18/05/09
3612

Shtorm в сообщении #594312 писал(а):

Посмотреть на график функции двух переменных? Или угадать предел, как кажется где-то ewert советовал?

Я не помню этих задач и соответственно, методов. Зорич приводит 5 примеров, и все на несуществование предела. Демидовича у меня нет, поройтесь сами, если интересно.
«Посмотреть на график» — конечно, не метод доказательства. После «угадать» тоже должно следовать доказательство.
Оно может быть простым, и «олимпиадным», и, например, основываться на определении:

Someone в сообщении #53265 писал(а):

Давайте посмотрим определение двойного предела:

, если для каждого можно найти такое , что для всех и , удовлетворяющих условию , выполняется неравенство .

Для предела в бесконечно удалённой точке определение, видимо, надо адекватно подправить (или или функцию подменить, чтоб тот же предел в нуле считать.).

Полагаю, определения предела в конечной и бесконечной точке должны быть «изоморфны». Соответственно, у функции типа предела в бесконечности нет, поскольку упомянутая kw_artem «асимптота-плоскость» имеется далеко не на всех путях в бесконечность.

Я всё сказал, что об этом помнится, и что пришло в голову: лично мне неохота развивать старую и имеющуюся в нескольких вариантах тему в архивном разделе. Конкретную задачку, если нашлась, можно порешать в «Помогите решить»


   

                  

Shtorm 

 Re: Помогите найти предел функции двух переменных

11. 07.2012, 19:02 

14/02/10
4956

AKM в сообщении #594393 писал(а):

Соответственно, у функции типа предела в бесконечности нет, поскольку упомянутая kw_artem

«асимптота-плоскость» имеется далеко не на всех путях в бесконечность.

Касательно того, что она имеется не на всех путях в бесконечность мы достаточно долго мусолили с ИСН в соответствующей теме и почти пришли к выводу о том, что если она есть только в заданном направлении, то она просто — есть! Простая аналогия на плоскости: Функция имеет конечный предел и соответственно горизонтальную асимптоту при , но при этом не имеет ни конечного предела, ни соответственно горизонтальной асимптоты при . Следовательно если есть хотя бы в одном направлении значит просто — есть. Значит по аналогии и в пространстве — если есть хотя бы в одном направлении значит просто — есть.

AKM в сообщении #594393 писал(а):

Для предела в бесконечно удалённой точке определение, видимо, надо адекватно подправить…

Такое «подправленное определение» есть в Фихтенгольце.

AKM в сообщении #594393 писал(а):

Я всё сказал, что об этом помнится, и что пришло в голову: лично мне неохота развивать старую и имеющуюся в нескольких вариантах тему в архивном разделе. Конкретную задачку, если нашлась, можно порешать в «Помогите решить»

Вот и получается, что определение есть — а конкретной общей методики нет. (Не странно ли?) А спрашивается как решать конкретную задачу — если нет общего метода? Я потому и стал писать в старой теме, поскольку этих тем уже много, а мне же не нужно решать задачку для зачёта или для диссертации. Я просто хочу выработать общую методику. А мне один раз уже закрыли мою тему, мотивировав, что таких тем уже полно и дескать не надо плодить — пишите в уже созданных.


   

                  

Munin 

 Re: Помогите найти предел функции двух переменных

31.07.2012, 01:06 

Заслуженный участник

30/01/06
72407

AKM в сообщении #594393 писал(а):

Полагаю, определения предела в конечной и бесконечной точке должны быть «изоморфны». Соответственно, у функции типа предела в бесконечности нет, поскольку упомянутая kw_artem

«асимптота-плоскость» имеется далеко не на всех путях в бесконечность.

О «пределе в бесконечности» здесь и не спрашивается, условия и очевидно, различны. Думаю, это вы «с устатку».


   

                  

Показать сообщения за: Все сообщения1 день7 дней2 недели1 месяц3 месяца6 месяцев1 год Поле сортировки АвторВремя размещенияЗаголовокпо возрастаниюпо убыванию 
  Страница 1 из 1
 [ Сообщений: 15 ] 

Модераторы: Модераторы Математики, Супермодераторы



Кто сейчас на конференции

Сейчас этот форум просматривают: нет зарегистрированных пользователей


Вы не можете начинать темы
Вы не можете отвечать на сообщения
Вы не можете редактировать свои сообщения
Вы не можете удалять свои сообщения
Вы не можете добавлять вложения

Найти:

Предел и непрерывность функции двух переменных

Для функции двух (и большего числа) переменных вводится понятие предела функции и непрерывность, аналогично случаю функции одной переменной.

Введем понятие окрестности точки.

Пусть функция определена в некоторой окрестности точки , кроме, может быть, самой этой точки.

Определение 1.6. Число называется пределом функции при и (или, что то же самое, при  ), если для любого существует такое, что для всех и и, удовлетворяющих неравенству , выполняется неравенство . Записывают:

(1.1)

или

.

Из определения следует, что если предел существует, то он не зависит от пути, по которому стремится к (число таких направлений бесконечно). Определения бесконечно малых и бесконечно больших величин являющихся функциями двух переменных, аналогичны соответствующим определениям для функций одной переменной.

Геометрический смысл предела функции двух переменных состоит в следующем. Каково бы ни было число , найдется -окрестность точки , что во всех точках , отличных от , аппликаты соответствующих точек поверхности отличаются от числа по модулю меньше, чем на .

Пример 1.2. Найти предел .

Решение. Будем приближаться к по прямой , где  некоторое число. Тогда

.

Функция в точке предела не имеет, т.к. при разных значениях предел функции не одинаковый (функция имеет различные предельные значения).

Предел функции двух переменных обладает свойствами, аналогичными свойствам предела функции одной переменной.

Определение 1.7. Функция (или ) называется непрерывной в точке , если она:

  1. определена в этой точке и некоторой ее окрестности;

  2. имеет предел ;

  3. этот предел равен значению функции в точке , т. е.

или .

Функция, непрерывная в каждой точке некоторой области, называется непрерывной в этой области. Точки, в которых непрерывность нарушается (не выполняется хотя бы одно из условий непрерывности функции в точке), называются точками разрыва этой функции. Точки разрыва могут образовывать целые линии разрыва. Так, например, функция имеет линю разрыва .

Можно дать другое, равносильное приведенному выше, определение непрерывности функции в точке. Обозначим , . Значит, и . Величины и называются приращениями аргументов и . Тогда . Величина называется полным приращением функции в точке .

Определение 1.8. Функция называется непрерывной в точке , если полное приращение функции в этой точке стремится к нулю, когда приращения ее аргументов и стремятся к нулю, т.е.

.

Пользуясь определением непрерывности и теоремами о пределах, можно доказать, что арифметические операции над непрерывными функциями и построение сложной функции из непрерывных функций приводит к непрерывным функциям – подобные теоремы имели для функций одной переменной.

  1. Дифференцирование фнп

2.1. Частные производные фнп

Рассмотрим линию пересечения поверхности с плоскостью , параллельной плоскости . Так как в этой плоскости сохраняет постоянное значение, то вдоль кривой будет меняться только в зависимости от изменения . Дадим независимой переменной приращение , тогда получит приращение, которое называется частным приращением по и обозначают через (на рисунке отрезок ), так что

.

Аналогично, если сохраняет постоянное значение, а получает приращение

параллельной плоскости .

Наконец, придав аргументу приращение , а аргументу приращение , получим для новое приращение , которое называется полным приращением функции и определяется формулой

.

На рисунке изображено отрезком .

Надо отметить, что, вообще говоря, полное приращение не равно сумме частных приращений, т.е. .

Определение 2.1. Частной производной по от функции называется предел отношения частного приращения по к приращению при стремлении к нулю. Обозначается: . Тогда

. (2.1)

Определение 2.2. Частной производной по от функции называется предел отношения частного приращения по к приращению при стремлении к нулю. Обозначается: . Тогда

. (2.2)

Таким образом, частная производная функции нескольких (двух, трех и больше) переменных определяется как производная функции одной из этих переменных при условии постоянства значений остальных независимых переменных. Поэтому частные производные функции находят по формулам и правилам вычисления производных функции одной переменной (при этом соответственно или считаются постоянной величиной).

Геометрический смысл частных производных: частная производная численно равна тангенсу угла наклона касательной к сечению поверхности плоскостью ;

частная производная численно равна тангенсу угла наклона касательной к сечению поверхности плоскостью .

Пример 2.1. Для данной функции требуется найти частные производные и . Найти значения частных производных в точке :

.

Решение. Находим частные производные в общем виде:

, .

Находим значения частных производных в точке :

, .

Пример 2.2. Найти частные производные , , , для следующей функции:

.

Решение.

.

Двойной предел функции двух переменных

6.

1.1. Предел функции нескольких переменных. Повторные пределы.

f: R nR задана в некоторой окрестности точки M0, кроме, может быть, самой точки M0.

Определение. Число А называется пределом функции

(M может приближаться к М0 по любому пути).

и .

(M приближается к М0 соответственно по горизонтали и по вертикали).

Теорема о связи двойного и повторных пределов.

Если  двойной предел и пределы ,,

то  повторные пределы ,и равны двойному.

Замечание 1. Обратное утверждение не верно.

Пример. f (x, y) =

,.

Однако двойной предел =

не существует, так как в любой окрестности точки (0, 0) функция принимает и «далекие » от нуля значения, например, если x = y, то f (x, y) = 0,5.

при движении M к M0 по любой прямой, двойной предел может не существовать.

Пример. f (x, y) = ,M0 (0, 0). M (x, y)  M0 (0, 0)

вдоль осей x = 0 или y = 0 f (x, y) = 0 .

y = kx, k  0 

y = x 2 , 

Вывод: предел (двойной) не существует.

Пример нахождения предела.

f (x, y) = , M0 (0, 0).

Покажем, что число 0 есть предел функции в точке M0.

= ,

 – расстояние между точками М и M0.( воспользовались неравенством ,

которое следует из неравенств )

Зададим  > 0 и пусть  = 2.  2 y 2 .

Очередь просмотра

Очередь

  • Удалить все
  • Отключить

YouTube Premium

Хотите сохраните это видео?

  • Пожаловаться

Пожаловаться на видео?

Выполните вход, чтобы сообщить о неприемлемом контенте.

Понравилось?

Не понравилось?

Текст видео

Занятия и репетиторство по Skype. Facebook: http://facebook.com/matan.channel , ВКонтакте: http://vk.com/matan.channel , Viber: +7 (927) 74-69-502, WhatsApp: +7 (927) 74-69-502.

Что такое предел функции двух переменных, и почему при вычислении пределов функций двух переменных следует учитывать траекторию, по которой переменная точка приближается к своему предельному значению.

Когда мы вычисляем пределы в обычном понимании, то есть, пределы функций одной переменной, мы говорим, что переменная приближается (или стремится) к своему предельному значению, при этом функция ведет себя так-то (стремится к конечному значению, бесконечно растет и так далее).

То же самое происходит и в случае предела функции двух переменных, только в этом случае переменная точка может приближаться к предельному положению разными способами.

Действительно, у переменного числа есть только два направления: слева направо (в сторону убывания) и справа налево (в сторону возрастания). А у переменной точки таких направлений бесконечно много: слева направо, справа налево, по диагонали, по криволинейной траектории — как угодно.

Так вот, предел функции двух переменных существует, если предельное значение функции двух переменных не зависит от траектории, по которой переменная точка приближается к своему предельному значению.

В остальном вычисление пределов функций двух переменных мало отличается от вычисления пределов функций одной переменной: точно так же нужно раскрывать неопределенности, использовать эквивалентность бесконечно малых и так далее.

Просмотрите видео по теме «Предел функции двух переменных», затем перейдите к вопросам по теме «Предел функции двух переменных» и попробуйте самостоятельно вычислит предложенные вам пределы функций двух переменных, и, наконец, проверьте себя, просмотрев ответы на вопросы по теме «Предел функции двух переменных».

Тема «Предел функции двух переменных»: https://youtu.be/m3JkKP6KRNA
Вопросы по теме «Предел функции двух переменных»: https://youtu. be/ZC-BjM5SlA0
Ответы на вопросы по теме «Предел функции двух переменных»: https://youtu.be/_cl2cMNjo4M

Чтобы подробнее ознакомиться с темой «Предел функции двух переменных», перейдите на сайт проекта «Матан».

Кафедра: Высшая математика

по дисциплине «Высшая математика»

Тема: «Предел и непрерывность функций нескольких переменных»

Понятие функции одной переменной не охватывает все зависимости, существующие в природе. Даже в самых простых задачах встречаются величины, значения которых определяются совокупностью значений нескольких величин.

Для изучения подобных зависимостей вводится понятие функции нескольких переменных.

Понятие функции нескольких переменных

Определение. Величина u называется функцией нескольких независимых переменных (x , y , z , …, t ), если каждой совокупности значений этих переменных ставится в соответствие определенное значение величины u .

Если переменная является функцией от двух переменных х и у , то функциональную зависимость обозначают

Символ f определяет здесь совокупность действий или правило для вычисления значения z по данной паре значений х и у .

при х = 1 и у = 1 имеем z = 4,

при х = 2 и у = 3 имеем z = 22,

при х = 4 и у = 0 имеем z = 16 и т.д.

Аналогично называется величина u функцией от трех переменных x , y , z , если дано правило, как по данной тройке значений x , y иz вычислить соответствующее значение u :

Здесь символ F определяет совокупность действий или правило для вычисления значения u , соответствующего данным значениям x , y иz .

при х = 2, у = -1 и z = -2 имеем u = -16 и т.д.

Таким образом, если в силу некоторого закона каждой совокупности п чисел (x , y , z , …, t ) из некоторого множества Е ставится в соответствие определенное значение переменной u , то и u называется функцией от п переменных x , y , z , …, t , определенной на множестве Е , и обозначается

Переменные x , y , z , …, t называются аргументами функции, множество Е – областью определения функции.

Областью определения функции называется множество всех значений аргументов, которым соответствуют какие-либо действительные значения функции.

Функция двух переменных z = f ( x , y ) в пространстве представляется некоторой поверхностью. То есть, когда точка с координатами х , у пробегает всю область определения функции, расположенную в плоскости хОу , соответствующая пространственная точка, вообще говоря, описывает поверхность.

Функцию трех переменных u = F ( x , y , z ) рассматривают как функцию точки некоторого множества точек трехмерного пространства. Аналогично, функцию п переменных u = f (x , y , z , …, t ) рассматривают как функцию точки некоторого п -мерного пространства.

Предел функции нескольких переменных

Для того чтобы дать понятие предела функции нескольких переменных, ограничимся случаем двух переменных х и у . По определению функция f ( x , y ) имеет предел в точке (х 0 , у 0 ), равный числу А , обозначаемый так:

(пишут еще f ( x , y )А при ( x , y ) → (х 0 , у 0 )), если она определена в некоторой окрестности точки (х 0 , у 0 ), за исключением, быть может, самой этой точки и если существует предел

Так же, как в случае функции одной переменной, можно ввести другое эквивалентное определение предела функции двух переменных: функция f имеет в точке (х 0 , у 0 ) предел, равный А , если она определена в некоторой окрестности точки (х 0 , у 0 ) за исключением, быть может, самой этой точки, и для любого ε > 0 найдется такое δ > 0, что

| f ( x , y )A | 0 найдется δ-окрестность точки (х 0 , у 0 ) такая, что для всех (x , y ) из этой окрестности, отличных от (х 0 , у 0 ), выполняется неравенство (3).

Так как координаты произвольной точки (x , y ) окрестности точки (х 0 , у 0 ) можно записать в виде х = х 0+ Δх , у = у 0 + Δу , то равенство (1) эквивалентно следующему равенству:

Рассмотрим некоторую функции, заданную в окрестности точки (х 0 , у 0 ), кроме, быть может, самой этой точки.

Пусть ω = (ωх , ωу ) – произвольный вектор длины единица (|ω| 2 = ωх 2 + ωу 2 = 1) и t > 0 – скаляр. Точки вида

Пример 2. Рассмотрим в R 2 функцию

Данная функция в точке (0, 0) на любой прямой y = kx , проходящей через начало координат, имеет предел, равный нулю:

Однако эта функция не имеет предела в точки (0, 0), ибо при у = х 2

коль скоро 0 0 найдется такое N > 0, что для всех х , у , для которых |x | > N , |y | > N , функция f определена и имеет место неравенство

Предел и непрерывность функции двух переменных.

— Студопедия

Поделись  


Раздел 4. Основы математического анализа.

Тема 4.3 Дифференциальное исчисление функции нескольких действительных переменных.

Основные понятия.

При рассмотрении многих вопросов из различных областей знания приходится изучать такие зависимости между переменными величинами, когда числовые значения одной из них полностью определяются значениями нескольких других.

Например, изучая физическое состояние какого-либо тела, приходится наблюдать изменение его свойств от точки к точке. Каждая точка тела задается тремя координатами: x, y, z. Поэтому, изучая, скажем, распределение плотности, заключаем, что плотность тела зависит от трех переменных: x, y, z. Если физическое состояние тела к тому же еще и меняется с течением времени t, то та же плотность будет зависеть уже от значений четырех переменных: x, y, z, t.

Другой пример: изучаются издержки производства на изготовление единицы некоторого вида продукции. Пусть:

x — затраты по материалам,

y — расходы на выплату заработной платы работникам,

z — амортизационные отчисления.

Очевидно, что издержки производства зависят от значений названных параметров x, y, z.

В задачах математического программирования обычно разыскивается экстремум (максимум или минимум) целевой функции при некоторых ограничениях на значения ее переменных, которые записываются в виде системы уравнений и неравенств. Обычно целевая функция являетсяфункцией нескольких переменных.

Определение.Если каждой совокупности значений n переменных из некоторого множества D этих совокупностей соответствует своё единственное значение переменной z, то говорят, что на множестве D задана функция n переменных.

Множество D называется областью определения или областью существования этой функции.

Рассмотрим функцию двух переменных, так как все основные понятия и теоремы, сформулированные для функций двух переменных, легко обобщаются на случай большего числа переменных.

Определение. Если каждой паре (x,y) значений двух независимых друг от друга переменных величин x и y из некоторого множества D соответствует единственное значение величины z, а каждому z соответствует хотя бы одна пара (x,y), то мы говорим, что z есть функция двух независимых переменных x и y, определенная в D.


Если рассматривается функция двух переменных, то совокупности чисел обозначаются, как правило, (x, y) и интерпретируются как точки координатной плоскости Oxy, а область определения функции z = f ( x, y ) двух переменных изобразится в виде некоторого множества точек на плоскости Oxy.

Так, например, областью определения функции является множество точек плоскости Oxy, координаты которых удовлетворяют соотношению

т. е. представляет собой круг радиуса r с центром в начале координат.

Для функции

областью определения служат точки, которые удовлетворяют условию т. е. внешние точки по отношению к заданному кругу.

Часто функции двух переменных задаются в неявном виде, т. е. как уравнение

, связывающее три переменные величины. В этом случае каждую из величин x, y, z можно рассматривать как неявную функцию двух остальных.

Если функции одной переменной соответствует определённая линия на плоскости (например, – всем знакомая школьная парабола), то график функции двух переменных располагается в трёхмерном пространстве. На практике чаще всего приходится иметь дело с поверхностью. Каждой паре ставится в соответствие точка принадлежащая графику функции и являющаяся концом перпендикуляра МР к плоскости Oxy.

 

Геометрическим изображением (графиком) функции двух переменных z = f ( x, y) является множество точек P ( x, y, z) в трехмерном пространстве Oxyz, координаты которых удовлетворяют уравнению z = f ( x, y ) (плоскости, цилиндры, шары, параболоиды и т.д.).


Графиком функции непрерывных аргументов, как правило, является некоторая поверхность в пространстве Oxyz, которая проектируется на координатную плоскость Oxy в область определения функции z= f ( x, y ).

Так, например, графиком функции является верхняя половина сферы, а графиком функции — нижняя половина сферы:

 

 

Графиком линейной функции z = ax + by + с является плоскость в пространстве Oxyz, а графиком функции z = сonst служит плоскость, параллельная координатной плоскости Oxyz.

Заметим, что функцию трех и большего числа переменных изобразить наглядно в виде графика в трехмерном пространстве невозможно.

Так же, как и при исследовании функций одной переменой, при рассмотрении функций нескольких переменных, важным моментом является отыскание области определения функции. Здесь полезно проводить некоторые аналогии, в частности, обращать особое внимание на те функции, в которых есть дроби, корни чётной степени, логарифмы и т. д.

Пример 1

Найти область определения функции .

Решение:

Так как знаменатель не может обращаться в ноль, то:

Уравнение задает на плоскости Oxy прямую, следовательно, областью определения заданной функции будет вся координатная плоскость Oxy, кроме точек, принадлежащих прямой .

Если бы по условию требовалось выполнить чертёж, то следовало бы изобразить координатную плоскость Oxy и пунктиром провести прямую Пунктир говорит о том, что линия не входит в область определения.

Пример 2

Найти область определения функции

Решение:

Подкоренное выражение должно быть неотрицательным:

Полученное неравенство задает полуплоскость, ограниченную снизу прямой

Пример 3. Найти и изобразить на чертеже область определения функции

Решение:

Подкоренное выражением должно быть неотрицательным и, учитывая, что знаменатель не может обращаться в ноль, получаем область определение, которая задается строгим неравенством:

Уравнение задает окружность радиуса с центром в начале координат, которая делит координатную плоскость на две части – внутреннюю и внешнюю области круга. Так как неравенство у нас строгое, то сама окружность не войдёт в область определения и поэтому её нужно провести пунктиром.

Теперь возьмем произвольную точку плоскости, не принадлежащуюокружности , и подставляем её координаты в неравенство .

Проще всего выбрать начало координат

Получено неверное неравенство, таким образом, точка не удовлетворяетнеравенству . Более того, данному неравенству не удовлетворяет и любая точка, лежащая внутри круга, и, стало быть, искомая область определения – внешняя его часть. Область определения традиционно штрихуется:

 

 

 

Пример 4.

Найти область определения функции

Решение:

Аргумент логарифма строго положителен, поэтому область определения задаётся системой

.

Неравенство задает на правую полуплоскость и исключает ось Oy.

Синус функция периодическая, промежутки знакопостоянства бесконечно чередуются и больше нуля синус на интервале .

Выполним чертёж, не забывая, что согласно первому условию, область определения ограничивается строго правой полуплоскостью.

 

Предел и непрерывность функции двух переменных.

Для функции двух (и большего числа) переменных вводится понятие предела функции и непрерывности, аналогично случаю функции одной переменной.

Введем понятие окрестности точки. Пусть на плоскости даны две точки и .

Определение.d (дельта)- окресностьюточки называется множество всех точек плоскости, расстояние от которых до точки меньше d,т.е. . Другими словами, d -окрестность точки — это все точки, лежащие внутри круга с центром и радиусом d.Обозначают d — окрестность точки символом

Определение. Число А называется пределом функции при и (или, что тоже самое, при ), если для любого существует такое , что для всех из d -окрестности точки причем выполняется неравенство .

Предел функции обозначается: . Т.е.

Нахождение предела функции двух переменных задача значительно более сложная, чем нахождение предела функции одной переменной. Задача усложняется из-за того, что для функции двух переменных на плоскости существует бесконечное множество направлений, по которым точка может приближаться к точке В случае функции одной переменной таких направлений всего два: слева и справа. Нахождение некоторых пределов упрощается, если появляется возможность с помощью замены перейти к функции одной переменной.

Пример. Найти

Решение.

Положим . Очевидно, что если и , то

Тогда

 

Предел функции двух переменных обладает свойствами, аналогичными свойствам предела функции одной переменной.

Определение. Функция называется непрерывной в точке если предел функции в этой точке существует и равен значению функции в этой точке:

Иначе говоря, функция непрерывна в точке если она:

1. определена в этой точке и некоторой ее окрестности;

2. существует конечный предел ;

3. выполнено равенство

Функция непрерывна в области,если она непрерывна во всех точках этой области. Точки, в которых непрерывность нарушается (не выполняется хотя бы одно из условий непрерывности функции в точке, а именно в этой точке либо функция не определена, либо не существует предела, либо значение функции не равно значению предела) называются точками разрыва этой функции. Например, для функции точками разрыва являются точки прямой (их называют линиями разрыва).

Для функций двух переменных справедливы теоремы о непрерывных функциях одной переменной.

Свойство 1. Если функция определена и непрерывна в замкнутой и ограниченной области D, то в этой области найдется, по крайней мере, одна точка , такая, что для остальных точек верно неравенство:

а также точка , такая, что для всех остальных точек верно неравенство:

Тогда – наибольшее значение функции, а – наименьшее значение функции в области D.

Т.е. непрерывная функция в замкнутой и ограниченной области D достигает, по крайней мере, один раз наибольшего значения и один раз наименьшего.

Свойство 2. Если функция определена и непрерывна в замкнутой ограниченной области D, то она принимает в области D все промежуточные значения между M и m. Следствием этого свойства может служить заключение, что если числа M и m разных знаков, то в области D функция, по крайней мере, один раз обращается в ноль.

Свойство 3. Функция , непрерывная в замкнутой ограниченной области D, ограничена в этой области, если существует такое число К, что для всех точек области верно неравенство

Свойство 4. Если функция определена и непрерывна в замкнутой ограниченной области D, то она равномерно непрерывна в этой области, т.е. для любого положительного числа e существует такое число , что для любых двух точек и области, находящихся на расстоянии, меньшем d, выполнено неравенство:









Лекции функции нескольких переменных

    Скачать с Depositfiles 

 

Лекции 1-4

ФУНКЦИИ НЕСКОЛЬКИХ ПЕРЕМЕННЫХ.

Контрольные вопросы.

  1. Частное и полное приращение функции нескольких переменных (ФНП).

  2. Предел функции нескольких переменных. Свойства пределов ФНП.

  3. Непрерывность ФНП. Свойства непрерывных функций.

  4. Частные производные первого порядка. 

Определение: если каждой рассматриваемой совокупности значений переменных  соответствует определенное значение переменной w, то будем называть wфункцией независимых переменных :

(1)

Определение: областью определения D(fфункции (1) называется совокупность таких наборов чисел , при которых определена функция (1).

Область D(f) может быть открытой или замкнутой. Например для функции:

D(f) будут все точки пространства, для которых выполняется неравенство  (замкнутый шар), а для функции (открытый шар).

В дальнейшем мы будем рассматривать в основном функции двух переменных, т.к. во-первых, принципиального различия между двумя и большим числом переменных нет, увеличение числа переменных ведет лишь к громоздкости выкладок. Во-вторых, случай двух переменных допускает наглядную геометрическую интерпретацию.

Геометрическим изображением функции двух переменных  является некоторая поверхность, которая может быть задана явно или неявно. Например: a)  — явное задание (параболоид вращения), б)  — неявное задание (сфера).

При построении графика функции часто пользуются методом сечений.

Пример. Построить график функции . 
Воспользуемся методом сечений.

– в плоскости – парабола.

 – в плоскости –парабола.

 – в плоскости – окружность.

Искомая поверхность – параболоид вращения.

Расстоянием между двумя произвольными точками  и  (евклидова) пространства  называется число

.

Множество точек  называется открытым кругом радиуса  с центром в точке ,  – окружностью радиуса  с центром в точке .

Открытый круг радиуса  с центром в точке  называется -окрестностью точки .

О
пределение. Точка  называется внутренней точкой множества , если существует -окрестность  точки , целиком принадлежащая множеству  (т.е.).

Определение. Точка  называется граничной точкой множества , если в любой ее -окрестности содержатся точки, как принадлежащие множеству , так и не принадлежащие ему.


Граничная точка множества может как принадлежать этому множеству, так и не принадлежать ему.

Определение. Множество  называется открытым, если все его точки – внутренние.

Определение. Множество  называется замкнутым, если оно содержит все свои граничные точки. Множество всех граничных точек множества  называется егограницей (и часто обозначается символом ). Заметим, что множество  является замкнутым и называется замыканием множества .

Пример. Если , то . При этом .

Частное и полное приращение функции.

Если одна независимая переменная (например, х) получает приращение х, а другая переменная не меняется, то функция получает приращение:

,

которое называется частным приращением функции  по аргументу х.

Если же все переменные получают приращения, то функция получает полное приращение:

Например, для функции  будем иметь:

; ;

.

Предел функции нескольких переменных.

Определение. Будем говорить, что последовательность точек  сходится при  к точке , если  при .

В этом случае точку  называют пределом указанной последовательности и пишут:  при .

Легко показать, что  тогда и только тогда, когда одновременно ,  (т.е. сходимость последовательности точек пространства эквивалентна покоординатной сходимости).

Определение. Число  называют пределом функции  при , если для   такое, что , как только.

В этом случае пишут  или  при .

При кажущейся полной аналогии понятий предела функций одной и двух переменных существует глубокое различие между ними. В случае функции одной переменной для существования предела в точке необходимо и достаточно равенство лишь двух чисел – пределов по двум направлениям: справа и слева от предельной точки . Для функции двух переменных стремление к предельной точке  на плоскости  может происходить по бесконечному числу направлений (и необязательно по прямой), и потому требование существования предела у функции двух (или нескольких) переменных «жестче» по сравнению с функцией одной переменной.

Пример. Найти .

Пусть стремление к предельной точке  происходит по прямой . Тогда .

Предел, очевидно, не существует, так как число  зависит от .

Свойства пределов ФНП:

Если существуют  и , то:

1) ;

2) ;

3) ,

где предельная точка  может быть конечной или бесконечной.

Непрерывность функции нескольких переменных

Определение. Говорят, что функция  непрерывна в точке , если

, т. е. .

Говорят, что функция  непрерывна в некоторой области, если она непрерывна в каждой точке этой области.

Свойства непрерывных функций: Если функции  и  непрерывны в точке , то этим же свойством обладают функции , , а если, то и функция .

Частные производные первого порядка.

Пусть функция  определена в области  и . Тогда при малых  определено ее частное приращение по : .

Определение. Частной производной функции  по переменной  в точке  называют предел

,

если он существует.

Частную производную по  обозначают одним из следующих символов:

.

Аналогично определяется частная производная по  и вводятся ее обозначения.

Легко видеть, что частная производная – это производная функции одной переменной, когда значение другой переменной фиксировано. Поэтому частные производные вычисляются по тем же правилам, что и вычисление производных функций одной переменной.

Пример. Найти частные производные функции .

Имеем: , . 

Исчисление III — Пределы

Показать мобильное уведомление Показать все примечания Скрыть все примечания

Уведомление для мобильных устройств

Похоже, вы используете устройство с «узкой» шириной экрана ( т.е. вы наверное на мобильном телефоне). Из-за характера математики на этом сайте лучше всего просматривать в ландшафтном режиме. Если ваше устройство не находится в ландшафтном режиме, многие уравнения будут отображаться сбоку вашего устройства (должна быть возможность прокрутки, чтобы увидеть их), а некоторые пункты меню будут обрезаны из-за узкой ширины экрана.

Раздел 2-1: Ограничения

В этом разделе мы рассмотрим ограничения, включающие функции более чем одной переменной. На самом деле мы сконцентрируемся в основном на пределах функций двух переменных, но идеи можно распространить и на функции с более чем двумя переменными. 9- }} f\влево( х \вправо)\]

— это левосторонний предел, который требует, чтобы мы рассматривали только те значения \(x\), которые меньше \(a\).

Другими словами, мы будем иметь \(\mathop {\lim }\limits_{x \to a} f\left( x \right) = L\) при условии \(f\left( x \right)\) приближается к \(L\), когда мы приближаемся к \(x = a\) (не пропуская \(x = a\)) с обеих сторон.

Теперь обратите внимание, что в этом случае есть только два пути, по которым мы можем двигаться по направлению к \(x = a\). Мы можем двигаться либо слева, либо справа. Тогда для существования предела функции одной переменной функция должна приближаться к одному и тому же значению по мере того, как мы идем по каждому из этих путей в направлении \(x = a\).

С функциями двух переменных нам придется сделать что-то подобное, за исключением того, что на этот раз (потенциально) потребуется гораздо больше работы. Давайте сначала обратимся к обозначениям и почувствуем, что мы собираемся просить в таких ограничениях.

Мы попросим взять предел функции \(f\left( {x,y} \right)\) при приближении \(x\) к \(a\) и при приближении \(y\) к \ (б\). Это можно записать несколькими способами. Вот пара более стандартных обозначений.

\[\ mathop {\lim }\limits_{x \to a\top y\to b} f\left({x,y} \right)\hspace{0.5in}\mathop {\lim }\limits_{\ влево( {x,y} \right) \to \left( {a,b} \right)} f\left( {x,y} \right)\]

В этом курсе мы чаще будем использовать второе обозначение. Второе обозначение также немного более полезно для иллюстрации того, что мы на самом деле делаем здесь, когда берем предел. Беря предел функции двух переменных, мы на самом деле спрашиваем, что делает значение \(f\left( {x,y} \right)\) при перемещении точки \(\left( {x,y } \right)\) все ближе и ближе к точке \(\left( {a,b} \right)\) фактически не позволяя ей быть \(\left( {a,b} \right)\).

Как и в случае с пределами функций одной переменной, для того, чтобы этот предел существовал, функция должна приближаться к одному и тому же значению независимо от пути, по которому мы движемся по направлению к \(\left( {a,b} \Правильно)\). Проблема, с которой мы сразу же сталкиваемся, заключается в том, что существует буквально бесконечное количество путей, по которым мы можем двигаться по направлению к \(\left( {a,b} \right)\). Вот несколько примеров путей, по которым мы могли бы пойти.

Мы добавили пару прямых путей, а также пару «странных» путей, которые не являются прямыми путями. Кроме того, мы включили здесь только 6 путей, и, как вы можете видеть, просто изменяя наклон прямых путей, их бесконечное количество, и тогда нам нужно будет рассмотреть пути, которые не являются прямыми путями.

Другими словами, чтобы показать, что предел существует, нам технически нужно проверить бесконечное количество путей и убедиться, что функция приближается к одному и тому же значению независимо от пути, который мы используем для приближения к точке.

К счастью для нас, мы можем использовать одну из основных идей из пределов Исчисления I, чтобы помочь нам установить пределы здесь.

Определение

Функция \(f\left( {x,y} \right)\) является непрерывной в точке \(\left( {a,b} \right)\), если

\[\ mathop {\lim }\limits_{\left({x,y} \right) \to \left({a,b} \right)} f\left({x,y} \right) = f \влево( {а,б} \вправо)\]

С графической точки зрения это определение означает то же самое, что и когда мы впервые увидели непрерывность в исчислении I. Функция будет непрерывной в точке, если в этой точке на графике нет дыр или разрывов.

Как это может помочь нам установить ограничения? Что ж, как и в исчислении I, если вы знаете, что функция непрерывна в точке \(\left( {a,b} \right)\), то вы также знаете, что

\[\ mathop {\lim }\limits_{\left({x,y} \right) \to \left({a,b} \right)} f\left({x,y} \right) = f \влево( {а,б} \вправо)\]

должно быть правдой. Итак, если мы знаем, что функция непрерывна в точке, то все, что нам нужно сделать, чтобы получить предел функции в этой точке, — это подставить точку в функцию.

Все стандартные функции, которые, как мы знаем, являются непрерывными, остаются непрерывными, даже если мы сейчас подставляем более одной переменной. Нам просто нужно следить за делением на ноль, квадратными корнями из отрицательных чисел, логарифмами нуля или отрицательных чисел, и т. д.

Обратите внимание, что идея о путях — это то, что мы не должны забывать, поскольку это хороший способ определить, не существует ли предел. Если мы сможем найти два пути, на которых функция приближается к разным значениям по мере приближения к точке, мы будем знать, что предела не существует.

Давайте рассмотрим пару примеров.

Пример 1. Определите, существуют ли следующие ограничения. Если они существуют, укажите значение предела. 92}\left( { — 1} \right) + \left( 1 \right)\left( 2 \right)\cos \left( {2\pi + \pi } \right) = — 14\]

б \(\displaystyle \mathop {\lim}\limits_{\left({x,y} \right) \to \left({5,1} \right)} \frac{{xy}}{{x + y}}\) Показать решение

В этом случае функция не будет непрерывной вдоль прямой \(y = — x\), так как при этом мы получим деление на ноль. Однако для этой проблемы нам не о чем беспокоиться, поскольку точка, в которой мы берем предел, не находится на этой линии.

Следовательно, все, что нам нужно сделать, это подставить точку, так как функция в этой точке непрерывна.

\[\ mathop {\lim }\limits_{\left({x,y} \right) \to \left({5,1} \right)} \frac{{xy}}{{x + y}} = \фракция{5}{6}\]

В предыдущем примере не было никаких пределов. Функции были непрерывны в рассматриваемой точке, поэтому все, что нам нужно было сделать, это подключить точку. Это, конечно, не всегда так, поэтому давайте рассмотрим несколько примеров, более типичных для тех, что вы здесь увидите. 92}}}\]

Показать решение

В этом случае функция не является непрерывной в рассматриваемой точке (явное деление на ноль). Однако это не означает, что ограничение невозможно. Мы видели много примеров этого в исчислении I, где функция не была непрерывной в точке, на которую мы смотрели, и все же предел существовал.

В случае этого предела обратите внимание, что мы можем разложить как числитель, так и знаменатель функции следующим образом: 92}}} = \mathop {\lim }\limits_{\left( {x,y} \right) \to \left({1,1} \right)} \frac{{\left({2x + y } \right)\left( {x — y} \right)}}{{\left( {x — y} \right)\left( {x + y} \right)}} = \mathop {\lim } \limits_{\left( {x,y} \right) \to \left({1,1} \right)} \frac{{2x + y}}{{x + y}}\]

Итак, как мы видели во многих примерах из Исчисления I, после разложения на множители и сокращения общих множителей мы приходим к функции, для которой фактически можно взять предел. Итак, чтобы закончить этот пример, все, что нам нужно сделать, это взять предел. 92}}} = \mathop {\lim }\limits_{\left( {x,y} \right) \to \left({1,1} \right)} \frac{{2x + y}}{{ х + у}} = \ гидроразрыва {3} {2} \]

Прежде чем мы перейдем к следующему набору примеров, мы должны отметить, что ситуация в предыдущем примере — это то, что обычно происходит во многих предельных примерах/задачах в исчислении I. Однако в исчислении III это, как правило, исключение в примерах /problems, как будет показано в следующем наборе примеров. Другими словами, не ожидайте, что большинство этих типов ограничений будут просто факторизованы, а затем будут существовать, как в исчислении I. 94}}}\) Показать решение

В этом случае функция не является непрерывной в рассматриваемой точке, поэтому мы не можем просто подключить точку. Также обратите внимание, что, в отличие от предыдущего примера, мы не можем факторизовать эту функцию и сделать некоторую отмену, чтобы можно было взять предел.

Следовательно, поскольку функция не является непрерывной в этой точке и поскольку мы не можем произвести факторинг, есть по крайней мере шанс, что предела не существует. Если бы мы могли найти два разных пути к точке, дающей разные значения предела, мы бы знали, что предела не существует. Двумя наиболее распространенными путями для проверки являются оси \(x\) и \(y\), так что давайте попробуем их.

Перед тем, как сделать это, нам нужно выяснить, что именно мы имеем в виду, когда говорим, что собираемся приблизиться к точке на пути. Когда мы приближаемся к точке на пути, мы будем делать это, либо фиксируя \(x\) или \(y\), либо связывая \(x\) и \(y\) через некоторую функцию. Таким образом, мы можем уменьшить предел до предела, включающего одну переменную, что мы знаем, как это сделать из исчисления I.

Итак, давайте посмотрим, что происходит вдоль оси \(x\). Если мы собираемся приблизиться к \(\left( {0,0} \right)\) по оси \(x\), мы можем воспользоваться тем фактом, что вдоль оси \(x\) мы знаем что \(y = 0\). Это означает, что по оси \(x\) мы подставим \(y = 0\) в функцию, а затем возьмем предел, когда \(x\) приблизится к нулю. 94}}} = \mathop {\lim }\limits_{\left({0,y}\right) \to \left({0,0}\right)} 0 = 0\]

Итак, одинаковый лимит по двум путям. Не поймите это неправильно. Это НЕ говорит о том, что предел существует и имеет нулевое значение. Это означает только то, что предел имеет одинаковое значение на двух путях.

Давайте рассмотрим третий довольно распространенный путь. 2}}}\) Показать решение 93}} \right) \to \left( {0,0} \right)} \frac{1}{2} = \frac{1}{2}\]

Теперь у нас есть два пути, которые дают разные значения предела, поэтому предела не существует.

Как показало нам это ограничение, мы можем и часто должны использовать пути, отличные от линий, как мы это делали в первой части этого примера.

Итак, как мы видели в предыдущем примере, пределы здесь немного отличаются от тех, которые мы видели в исчислении I. Ограничения по нескольким переменным довольно сложно оценить, и мы показали несколько примеров, где это потребовало небольшой работы. просто чтобы показать, что предела не существует.

14.2: Пределы и непрерывность — Mathematics LibreTexts

  1. Последнее обновление
  2. Сохранить как PDF
  • Идентификатор страницы
    4537
  • Цели обучения
    • Вычислить предел функции двух переменных.
    • Узнайте, как функция двух переменных может приближаться к разным значениям в граничной точке в зависимости от пути приближения.
    • Укажите условия непрерывности функции двух переменных.
    • Проверить непрерывность функции двух переменных в точке.
    • Вычислить предел функции трех и более переменных и проверить непрерывность функции в точке.

    Теперь мы рассмотрели функции более чем одной переменной и увидели, как их изобразить. В этом разделе мы увидим, как взять предел функции более чем одной переменной и что означает непрерывность функции более чем одной переменной в точке ее области определения. Оказывается, у этих концепций есть аспекты, которые просто не встречаются с функциями одной переменной.

    Предел функции двух переменных

    Напомним из раздела 2.5, что определение предела функции одной переменной:

    Пусть \(f(x)\) определено для всех \(x≠a\) в открытом интервале, содержащем \(a\). Пусть \(L\) — действительное число. Тогда

    \[\lim_{x→a}f(x)=L \nonumber \]

    , если для любого \(ε>0,\) существует такое \(δ>0\), что если \ (0<|x−a|<δ\) для всех \(x\) в области определения \(f\), тогда

    \[|f(x)−L|<ε. \номер\] 92\} \nonumber \]

    , как показано на рисунке \(\PageIndex{1}\).

    Рисунок \(\PageIndex{1}\): диск \(δ\) с центром вокруг точки \((2,1)\).

    Идея круга \(δ\) появляется в определении предела функции двух переменных. Если \(δ\) мало, то все точки \((x,y)\) в круге \(δ\) близки к \((a,b)\). Это совершенно аналогично тому, как х близко к а в определении предела функции одной переменной. В одном измерении мы выражаем это ограничение как

    \[a−δ

    В более чем одном измерении мы используем диск \(δ\).

    Определение: предел функции двух переменных

    Пусть \(f\) — функция двух переменных, \(x\) и \(y\). Предел \(f(x,y)\) при приближении \((x,y)\) к \((a,b)\) равен \(L\), записанному

    \[\lim_{(x ,y)→(a,b)}f(x,y)=L \nonumber \]

    , если для каждого \(ε>0\) существует достаточно малое \(δ>0\) такое, что для всех точек \((x,y)\) в диске \(δ\) вокруг \((a,b)\), за исключением, возможно, самого \((a,b)\), значение \(f( x,y)\) не более чем на \(ε\) от \(L\) (рисунок \(\PageIndex{2}\)). 92}<δ. \nonumber \]

    Рисунок \(\PageIndex{2}\): Предел функции с двумя переменными требует, чтобы \(f(x,y)\) находилось в пределах \(ε\) от \(L\) всякий раз, когда \((x,y)\) находится в пределах \(δ\) от \((a,b)\). Чем меньше значение \(ε\), тем меньше значение \(δ\).

    Доказательство существования предела с использованием определения предела функции двух переменных может быть сложной задачей. Вместо этого мы воспользуемся следующей теоремой, которая поможет нам найти пределы. Формулы этой теоремы являются расширением формул теоремы о предельных законах из книги «Предельные законы».

    Предельные законы для функций двух переменных

    Пусть \(f(x,y)\) и \(g(x,y)\) определены для всех \((x,y)≠(a,b) \) в окрестности \((a,b)\), и предположим, что эта окрестность полностью содержится внутри области определения \(f\). Предположим, что \(L\) и \(M\) — действительные числа такие, что

    \[\lim_{(x,y)→(a,b)}f(x,y)=L \nonumber \]

    и

    \[\lim_{(x,y)→(a,b)}g(x,y)=M, \nonumber \]

    и пусть \(c\) — константа. Тогда верно каждое из следующих утверждений:

    Постоянный закон:

    \[\lim_{(x,y)→(a,b)}c=c \nonumber \]

    Законы тождества:

    \[\lim_{(x, y)→(a,b)}x=a \nonumber \]

    \[\lim_{(x,y)→(a,b)}y=b \nonumber \]

    Закон суммы:

    \[\lim_{(x,y)→(a,b)}(f(x,y)+g(x,y))=L+M \nonumber \]

    Разностный закон:

    \[\lim_{(x,y)→(a,b)}(f(x,y)−g(x,y))=L−M \nonumber \]

    Постоянный кратный закон:

    9n \nonumber \]

    для любого положительного целого числа \(n\).

    Корневой закон:

    \[\lim_{(x,y)→(a,b)}\sqrt[n]{f(x,y)}=\sqrt[n]{L} \nonumber \]

    для всех \(L\), если \(n\) нечетно и положительно, и для \(L≥0\), если n четно и положительно.

    Доказательства этих свойств аналогичны доказательствам для пределов функций одной переменной. Мы можем применить эти законы для нахождения пределов различных функций.

    Пример \(\PageIndex{1}\): нахождение предела функции двух переменных 92−4(2)+3(−1)−6 \\[4pt] =−6. \end{выравнивание*}\]

    б. Перед применением частного закона нам нужно убедиться, что предел знаменателя отличен от нуля. Используя разностный закон, постоянный множественный закон и закон тождества,

    \[\begin{align*} \lim_{(x,y)→(2,−1)}(4x−3y) =\lim_{(x ,y)→(2,−1)}4x−\lim_{(x,y)→(2,−1)}3y \\[4pt] =4(\lim_{(x,y)→(2, −1)}x)−3(\lim_{(x,y)→(2,−1)}y) \\[4pt] =4(2)−3(−1)=11. \end{align*}\]

    Поскольку предел знаменателя не равен нулю, применяется закон частного. Теперь вычислим предел числителя, используя разностный закон, закон констант-множителей и закон тождества:

    \[\begin{align*} \lim_{(x,y)→(2,−1)}(2x+3y) =\lim_{(x,y)→(2,−1)}2x+\ lim_{(x,y)→(2,−1)}3y \\[4pt] =2(\lim_{(x,y)→(2,−1)}x)+3(\lim_{(x ,y)→(2,−1)}y) \\[4pt] =2(2)+3(−1)=1. \end{align*}\]

    Следовательно, согласно закону о частных имеем

    \[\begin{align*} \lim_{(x,y)→(2,−1)}\dfrac{2x+ 3y}{4x−3y} =\dfrac{\displaystyle \lim_{(x,y)→(2,−1)}(2x+3y)}{\displaystyle \lim_{(x,y)→(2, −1)}(4x−3y)} \\[4pt] =\dfrac{1}{11}. 2}=0 \номер\] 92}=\tfrac{1}{2}. \nonumber \]

    Это верно для любой точки на прямой \(y=x\). Если мы позволим \(x\) приблизиться к нулю, оставаясь на этой линии, значение функции останется фиксированным на \(\tfrac{1}{2}\), независимо от того, насколько мал \(x\).

    Выберите значение ε меньше \(1/2\) — скажем, \(1/4\). Тогда, независимо от того, насколько маленький диск \(δ\) мы нарисуем вокруг \((0,0)\), значения \(f(x,y)\) для точек внутри этого диска \(δ\) будут включают как \(0\), так и \(\tfrac{1}{2}\). Следовательно, определение предела в точке никогда не выполняется, и предел не существует. 92}.\) Вдоль прямой \(y=0\) функция равна нулю; вдоль линии \(y=x\) функция равна \(\tfrac{1}{2}\).

    б. Подобно а., мы можем приблизиться к началу координат по любой прямой, проходящей через начало координат. Если мы попытаемся использовать ось \(x\) (т. е. \(y=0\)), то функция останется фиксированной на нуле. То же верно и для оси \(y\). Предположим, мы приближаемся к началу координат по прямой линии наклона \(k\). Уравнение этой линии \(y=kx\). Тогда предел становится

    92\) (Рисунок \(\PageIndex{4}\)).

    1. Точка \(P_0\) называется внутренней точкой \(S\), если существует диск \(δ\) с центром вокруг \(P_0\), полностью содержащийся в \(S\).
    2. Точка \(P_0\) называется граничной точкой \(S\), если каждый \(δ\) круг с центром в \(P_0\) содержит точки как внутри, так и снаружи \(S\).
    Рисунок \(\PageIndex{4}\): в показанном множестве \(S\) \((−1,1)\) является внутренней точкой, а \((2,3)\) является граничной точкой . 92\) (Рисунок \(\PageIndex{4}\)).

    1. \(S\) называется открытым множеством , если каждая точка \(S\) является внутренней точкой.
    2. \(S\) называется замкнутым множеством , если оно содержит все свои граничные точки.

    Примером открытого множества является диск \(δ\). Если мы включим границу диска, то он станет замкнутым множеством. Множество, которое содержит некоторые, но не все, его граничные точки, не является ни открытым, ни закрытым. Например, если мы включаем половину границы круга \(δ\), но не другую половину, то множество не является ни открытым, ни закрытым. 92\) (Рисунок \(\PageIndex{4}\)).

    1. Открытое множество \(S\) является связным множеством , если оно не может быть представлено как объединение двух или более непересекающихся, непустых открытых подмножеств.
    2. Множество \(S\) является областью , если оно открыто, связно и непусто.

    Определение предела функции двух переменных требует, чтобы диск \(δ\) содержался внутри области определения функции. Однако если мы хотим найти предел функции в граничной точке области, \(δ\) диск не содержится внутри домена. По определению некоторые точки диска \(δ\) находятся внутри области, а некоторые — снаружи. Поэтому нам нужно рассматривать только точки, которые находятся как внутри круга \(δ\), так и внутри области определения функции. Это приводит к определению предела функции в граничной точке. 2}<δ. \номер\] 92} \номер\]

    Непрерывность функций двух переменных

    В разделе Непрерывность мы определили непрерывность функции одной переменной и увидели, как она зависит от предела функции одной переменной. В частности, необходимы три условия для непрерывности \(f(x)\) в точке \(x=a\)

    1. \(f(a)\).
    2. \(\displaystyle \lim_{x→a}f(x)\) существует.
    3. \(\displaystyle \lim_{x→a}f(x)=f(a).\)

    Эти три условия необходимы и для непрерывности функции двух переменных.

    Определение: непрерывные функции

    Функция \(f(x,y)\) непрерывна в точке \((a,b)\) своей области определения, если выполняются следующие условия:

    1. \(f (а,б)\) существует.
    2. \(\displaystyle \lim_{(x,y)→(a,b)}f(x,y)\) существует.
    3. \(\ displaystyle \ lim_ {(x, y) → (a, b)} f (x, y) = f (a, b). \)
    Пример \(\PageIndex{4}\): демонстрация непрерывности функции двух переменных

    Показать, что функция

    \[f(x,y)=\dfrac{3x+2y}{x+y+1} \nonumber \]

    непрерывна в точке \((5,−3). \)

    Решение

    Согласно определению непрерывности должны быть выполнены три условия. В этом примере \(a=5\) и \(b=−3.\)

    1. \(f(a,b)\) существует. Это верно, потому что область определения функции f состоит из тех упорядоченных пар, для которых знаменатель не равен нулю (т. Е. \ (x + y + 1 ≠ 0 \)). Точка \((5,−3)\) удовлетворяет этому условию. Кроме того,

    \[f(a,b)=f(5,−3)=\dfrac{3(5)+2(−3)}{5+(−3)+1}=\dfrac{15−6 {2+1}=3. \nonumber \]

    2. \(\displaystyle \lim_{(x,y)→(a,b)}f(x,y)\) существует. Это также верно:

    \[\begin{align*} \lim_{(x,y)→(a,b)}f(x,y) =\lim_{(x,y)→(5,− 3)} \ dfrac {3x + 2y} {x + y + 1} \\ = \ dfrac {\ displaystyle \ lim_ {(x, y) → (5, −3)} (3x + 2y)} {\ displaystyle \lim_{(x,y)→(5,−3)}(x+y+1)} \\ = \dfrac{15−6}{5−3+1} \\ = 3. \end{align *}\]

    3. \(\displaystyle \lim_{(x,y)→(a,b)}f(x,y)=f(a,b).\) Это верно, потому что мы только что показано, что обе части этого уравнения равны трем. 92}<δ\) верно, \(|f(x,y)−f(a,b)|<ε. \) Это определение можно комбинировать с формальным определением (то есть эпсилон–дельта определение ) непрерывности функции одной переменной для доказательства следующих теорем:

    Сумма непрерывных функций непрерывна

    Если \(f(x,y)\) непрерывна в \((x_0,y_0)\ ), и \(g(x,y)\) непрерывна в \((x_0,y_0)\), то \(f(x,y)+g(x,y)\) непрерывна в \(( х_0,у_0)\).

    Произведение непрерывных функций непрерывно 92\) в диапазон \(R⊆R.\) Предположим, что \(g\) непрерывна в некоторой точке \((x_0,y_0)∈D\) и определим \(z_0=g(x_0,y_0)\) . Пусть f будет функцией, которая отображает \(R\) в \(R\), так что \(z_0\) находится в области определения \(f\). Наконец, предположим, что \(f\) непрерывно в \(z_0\). Тогда \(f∘g\) непрерывен в \((x_0,y_0)\), как показано на рисунке \(\PageIndex{7}\).

    Рисунок \(\PageIndex{7}\): Композиция двух непрерывных функций непрерывна.

    Теперь воспользуемся предыдущими теоремами, чтобы показать непрерывность функций в следующих примерах. 3\)? Что значит быть непрерывным в точке в четырех измерениях? 92}<δ\большой\}. \nonumber \]

    Чтобы показать, что предел функции трех переменных существует в точке \((x_0,y_0,z_0)\), достаточно показать, что для любой точки шара \(δ\) с центром в \((x_0,y_0,z_0)\) значение функции в этой точке сколь угодно близко к фиксированному значению (предельное значение). Все предельные законы для функций двух переменных справедливы и для функций более чем двух переменных.

    Пример \(\PageIndex{6}\): нахождение предела функции трех переменных 92y−3z}{2x+5y−z}. \nonumber \]

    Решение

    Прежде чем мы сможем применить частное, нам нужно проверить, что предел знаменателя не равен нулю. Используя разностный закон, закон тождества и постоянный закон,

    \[\begin{align*}\lim_{(x,y,z)→(4,1,−3)}(2x+5y−z ) =2(\lim_{(x,y,z)→(4,1,−3)}x)+5(\lim_{(x,y,z)→(4,1,−3)}y )−(\lim_{(x,y,z)→(4,1,−3)}z) \\ = 2(4)+5(1)−(−3) \\ = 16. \end{ align*}\]

    Так как это не ноль, мы затем находим предел числителя. Используя закон произведения, степенной закон, разностный закон, постоянный множественный закон и закон тождества, 92}=2 \номер\]

    Основные понятия

    • Для изучения пределов и непрерывности функций двух переменных мы используем диск \(δ\) с центром в заданной точке.
    • Функция многих переменных имеет предел, если для любой точки шара \(δ\) с центром в точке \(P\) значение функции в этой точке сколь угодно близко к фиксированному значению (предельное значение ).
    • Предельные законы, установленные для функции одной переменной, имеют естественное распространение на функции более чем одной переменной.
    • Функция двух переменных непрерывна в точке, если в этой точке существует предел, функция существует в этой точке и предел и функция в этой точке равны.

    Глоссарий

    граничная точка
    точка \(P_0\) в \(R\) является граничной точкой, если каждый \(δ\) круг с центром в \(P_0\) содержит точки как внутри, так и вне \(R\)
    закрытый набор
    множество \(S\), содержащее все его граничные точки 93\), лежащие на расстоянии менее \(δ\) от \((x_0,y_0,z_0)\)
    внутренняя точка
    точка \(P_0\) множества \(\mathbb{R}\) является граничной точкой, если существует диск \(δ\) с центром в \(P_0\), полностью содержащийся в \(\mathbb{R}\ )
    открытый набор
    множество \(S\), не содержащее ни одной из своих граничных точек
    регион
    открытое связное непустое подмножество \(\mathbb{R}^2\)

    14. 2: Limits and Continuity распространяется по незаявленной лицензии и был создан, изменен и/или курирован LibreTexts.

    1. Наверх
      • Была ли эта статья полезной?
      1. Тип изделия
        Раздел или страница
        Показать страницу TOC
        нет
        Включено
        да
      2. Метки
          На этой странице нет тегов.

      исчисление — Показать, что предела не существует (две переменные)

      спросил

      Изменено 3 года, 7 месяцев назад

      Просмотрено 14 тысяч раз

      $\begingroup$

      Я только начал изучать исчисление нескольких переменных и их ограничения. Я тоже не силен в ограничениях.

      Я хочу ответить на этот вопрос:

      Показать, что следующий предел не существует

      92}=1$$

      Следовательно, лимита не существует.

      Это несколько правильно? Как лучше всего ответить на подобный вопрос?

      Кроме того, при демонстрации того, что этого предела не существует, нужно ли мне находить разные значения пределов как для ${x\to 0}$, так и для ${y\to 0}$? Или достаточно одного, например. если я просто найду два разных значения для двух пределов для ${x\to 0}$, вообще не используя ${y\to 0}$ в своих вычислениях, это нормально?

      Спасибо!

      • исчисление 92}$$ — предел по линии $y=0$.

        И последний предел, который вы вычислили, соответствует линии $y=x$.

        Итак, чтобы ответить на ваш вопрос, да, это было бы вполне приемлемо, если бы вы не вычисляли лимит по $y=0$. Просто показать два примера, где предел оказывается разным в разных направлениях, достаточно, чтобы показать, что предела не существует.

        $\endgroup$

        7

        $\begingroup$

        Ваши рассуждения в порядке. Если бы ограничение существовало, вы бы получили одно и то же значение для каждого ограничения направления, что было не так.

        $\endgroup$

        2

        $\begingroup$

        То, что вы сделали, абсолютно правильно, но, возможно, вы можете немного облегчить его следующим образом:

        Для $\;x=0\;,\;\;y\to0\;$ предел явно равен $\; 0\;$, тогда как для $\;x=y\;$ и $\;x\to0\;$ получаем 9{-2}}= \begin{случаи} 0 & m\ne 1 \\[4px] 1 и м=1 \end{случаи} $$ Поступая таким образом, можно сразу показать, как решить бизнес. Конечно, если вы обнаружите, что все пределы вдоль линий равны, вы не можете заключить, что предел существует; вместо этого вам следует попробовать другие кривые, если вы подозреваете, что предела не существует.

        $\endgroup$

        $\begingroup$

        То, что вы сделали, правильно, но обратите внимание, что после доказательства того, что предел равен $0$, когда вы принимаете $y=0$, тот факт, что он также равен $0$, когда вы принимаете $x=0$, не имеет значения. Здесь важно то, что переход к $(0,0)$ в двух разных направлениях приводит к двум различным ограничениям. Следовательно, нет (глобального) предела в $(0,0)$.

        $\endgroup$

        Твой ответ

        Зарегистрируйтесь или войдите в систему

        Зарегистрируйтесь с помощью Google

        Зарегистрироваться через Facebook

        Зарегистрируйтесь, используя электронную почту и пароль

        Опубликовать как гость

        Электронная почта

        Требуется, но никогда не отображается

        Опубликовать как гость

        Электронная почта

        Требуется, но не отображается

        Нажимая «Опубликовать свой ответ», вы соглашаетесь с нашими условиями обслуживания, политикой конфиденциальности и политикой использования файлов cookie

        .

        4.2 Пределы и непрерывность. Расчет, том 3

        Цели обучения

        • 4.2.1 Вычислить предел функции двух переменных.
        • 4.2.2 Узнайте, как функция двух переменных может приближаться к разным значениям в граничной точке в зависимости от пути приближения.
        • 4.2.3 Сформулируйте условия непрерывности функции двух переменных.
        • 4.2.4 Проверить непрерывность функции двух переменных в точке.
        • 4.2.5 Вычислить предел функции трех и более переменных и проверить непрерывность функции в точке.

        Теперь мы рассмотрели функции более чем одной переменной и увидели, как их изобразить. В этом разделе мы увидим, как взять предел функции более чем одной переменной и что означает непрерывность функции более чем одной переменной в точке ее области определения. Оказывается, у этих концепций есть аспекты, которые просто не встречаются с функциями одной переменной.

        Предел функции двух переменных

        Вспомните из «Предела функции» определение предела функции одной переменной:

        Пусть f(x)f(x) определена для всех x≠ax≠a в открытом интервале, содержащем п.в. Пусть LL — действительное число. Затем

        limx→af(x)=Llimx→af(x)=L

        , если для каждого ε>0,ε>0 существует такое δ>0,δ>0, что если 0<|x−a|<δ0<|x−a|<δ для всех xx в области ф, ф, затем

        |f(x)−L|<ε.|f(x)−L|<ε.

        Прежде чем мы сможем адаптировать это определение для определения предела функции двух переменных, нам сначала нужно увидеть, как расширить идею открытого интервала одной переменной до открытого интервала двух переменных.

        Определение

        Рассмотрим точку (a,b)∈ℝ2.(a,b)∈ℝ2. Диск δδ с центром в точке (a,b)(a,b) определяется как открытый диск радиуса δδ с центром в точке (a,b)(a,b), то есть

        {(x,y )∈ℝ2|(x−a)2+(y−b)2<δ2}{(x,y)∈ℝ2|(x−a)2+(y−b)2<δ2}

        , как показано на следующий график.

        Рисунок 4.14 Диск δδ с центром вокруг точки (2,1).(2,1).

        Идея δδ-диска возникает в определении предела функции двух переменных. Если δδ мало, то все точки (x,y)(x,y) в круге δδ близки к (a,b).(a,b). Это совершенно аналогично тому, что хх близок к аа в определении предела функции одной переменной. В одном измерении мы выражаем это ограничение как

        а-δ

        В более чем одном измерении мы используем диск δδ.

        Определение

        Пусть ff — функция двух переменных, xx и y.y. Предел f(x,y)f(x,y) при приближении (x,y)(x,y) к (a,b)(a,b) равен L,L, записанному как

        lim(x,y )→(a,b)f(x,y)=Llim(x,y)→(a,b)f(x,y)=L

        , если для каждого ε>0ε>0 существует достаточно малое δ >0δ>0 такое, что для всех точек (x,y)(x,y) в δδ-диске вокруг (a,b),(a,b), за исключением, возможно, самой (a,b)(a,b) , значение f(x,y)f(x,y) удалено от LL не более чем на εε (рис. 4.15). Используя символы, запишем следующее: Для любого ε>0,ε>0 существует число δ>0δ>0 такое, что

        |f(x,y)−L|<εwhenever0<(x−a)2+(y−b)2<δ. |f(x,y)−L|<εwhenever0<(x−a)2 +(y−b)2<δ.

        Рисунок 4.15 Предел функции с двумя переменными требует, чтобы f(x,y)f(x,y) находилась в пределах εε от LL всякий раз, когда (x,y)(x,y) находится в пределах δδ от (a,b).(a ,б). Чем меньше значение ε,ε, тем меньше значение δ.δ.

        Доказательство существования предела с помощью определения предела функции двух переменных может быть сложной задачей. Вместо этого мы воспользуемся следующей теоремой, которая поможет нам найти пределы. Формулы этой теоремы являются расширением формул теоремы о предельных законах из книги «Предельные законы».

        Теорема 4.1

        Предельные законы для функций двух переменных

        Пусть f(x,y)f(x,y) и g(x,y)g(x,y) определены для всех (x,y)≠(a,b)(x,y)≠( a,b) в окрестности (a,b),(a,b) и пусть эта окрестность полностью содержится внутри области определения ф.ф. Предположим, что LL и MM — вещественные числа такие, что lim(x,y)→(a,b)f(x,y)=Llim(x,y)→(a,b)f(x,y)=L и lim(x,y)→(a,b)g(x,y)=M,lim(x,y)→(a,b)g(x,y)=M, и пусть cc — константа. Тогда верно каждое из следующих утверждений:

        Постоянный закон:

        lim(x,y)→(a,b)c=clim(x,y)→(a,b)c=c

        (4.2)

        Законы тождества:

        lim(x,y)→(a,b)x=alim(x,y)→(a,b)x=a

        (4.3)

        lim(x,y)→(a,b) y=blim(x,y)→(a,b)y=b

        (4.4)

        Закон суммы:

        lim(x,y)→(a,b)(f(x,y) +g(x,y))=L+Mlim(x,y)→(a,b)(f(x,y)+g(x,y))=L+M

        (4.5)

        Разностный закон:

        lim(x,y)→(a,b)(f(x,y)−g(x,y))=L−Mlim(x,y)→(a,b)(f (x,y)−g(x,y))=L−M

        (4.6)

        Постоянный кратный закон:

        lim(x,y)→(a,b)(cf(x,y))=cLlim(x,y)→(a,b)(cf (x,y))=cL

        (4.7)

        Закон произведения:

        lim(x,y)→(a,b)(f(x,y)g(x,y))=LMlim (x,y)→(a,b)(f(x,y)g(x,y))=LM

        (4. 8)

        Частный закон:

        lim(x,y)→(a ,b)f(x,y)g(x,y)=LMforM≠0lim(x,y)→(a,b)f(x,y)g(x,y)=LMforM≠0

        (4.9 )

        Степенной закон:

        lim(x,y)→(a,b)(f(x,y))n=Lnlim(x,y)→(a,b)(f(x,y) ))n=Ln

        (4.10)

        для любого натурального числа n.n.

        Корневой закон:

        lim(x,y)→(a,b)f(x,y)n=Lnlim(x,y)→(a,b)f(x,y)n=Ln

        (4.11)

        для всех LL, если nn нечетное и положительное, и для L≥0L≥0, если nn четное и положительное, при условии, что f(x, y) ≥ 0f(x, y) ≥ 0 для всех ( x, y) ≠ (a, b)(x, y) ≠ (a, b) в окрестности (a, b)(a, b).

        Доказательства этих свойств аналогичны доказательствам для пределов функций одной переменной. Мы можем применить эти законы для нахождения пределов различных функций.

        Пример 4,8

        Нахождение предела функции двух переменных

        Нахождение каждого из следующих пределов:

        1. lim(x,y)→(2,−1)(x2−2xy+3y2−4x+3y−6)lim (x,y)→(2,−1)(x2−2xy+3y2−4x+3y−6)
        2. lim(x,y)→(2,−1)2x+3y4x−3ylim(x,y)→(2,−1)2x+3y4x−3y
        Решение
        1. Сначала используйте законы суммы и разности, чтобы разделить члены:

          lim(x,y)→(2,−1)(x2−2xy+3y2−4x+3y−6)=(lim(x,y)→ (2,−1)x2)−(lim(x,y)→(2,−1)2xy)+(lim(x,y)→(2,−1)3y2)−(lim(x,y) →(2,−1)4x)+(lim(x,y)→(2,−1)3y)−(lim(x,y)→(2,−1)6). lim(x,y) →(2,−1)(x2−2xy+3y2−4x+3y−6)=(lim(x,y)→(2,−1)x2)−(lim(x,y)→(2,− 1)2xy)+(lim(x,y)→(2,−1)3y2)−(lim(x,y)→(2,−1)4x)+(lim(x,y)→(2, −1)3y)−(lim(x,y)→(2,−1)6).


          Далее используйте закон множителей констант для второго, третьего, четвертого и пятого пределов:

          =(lim(x,y)→(2,−1)x2)−2(lim(x,y)→( 2,−1)xy)+3(lim(x,y)→(2,−1)y2)−4(lim(x,y)→(2,−1)x)+3(lim(x, y)→(2,−1)y)−lim(x,y)→(2,−1)6.=(lim(x,y)→(2,−1)x2)−2(lim(x ,y)→(2,−1)xy)+3(lim(x,y)→(2,−1)y2)−4(lim(x,y)→(2,−1)x)+3 (lim(x,y)→(2,−1)y)−lim(x,y)→(2,−1)6.


          Теперь используем степенной закон для первого и третьего пределов и закон произведения для второго предела:

          =(lim(x,y)→(2,−1)x)2−2(lim(x, y)→(2,−1)x)(lim(x,y)→(2,−1)y)+3(lim(x,y)→(2,−1)y)2−4(lim (x,y)→(2,−1)x)+3(lim(x,y)→(2,−1)y)−lim(x,y)→(2,−1)6.=( lim(x,y)→(2,−1)x)2−2(lim(x,y)→(2,−1)x)(lim(x,y)→(2,−1)y) +3(lim(x,y)→(2,−1)y)2−4(lim(x,y)→(2,−1)x)+3(lim(x,y)→(2, −1)y)−lim(x,y)→(2,−1)6.


          Наконец, используйте законы тождества для первых шести пределов и постоянный закон для последнего предела:
          lim(x,y)→(2,−1)(x2−2xy+3y2−4x+3y−6)= (2)2−2(2)(−1)+3(−1)2−4(2)+3(−1)−6=−6. lim(x,y)→(2,−1) (x2−2xy+3y2−4x+3y−6)=(2)2−2(2)(−1)+3(−1)2−4(2)+3(−1)−6=−6 .
        2. Прежде чем применять частное, нужно убедиться, что предел знаменателя отличен от нуля. Используя разностный закон, константно-кратный закон и закон тождества,

          lim(x,y)→(2,−1)(4x−3y)=lim(x,y)→(2,−1)4x−lim( х, у)→(2,−1)3у=4(lim(х,у)→(2,−1)х)−3(lim(х,у)→(2,−1)у)=4 (2)−3(−1)=11.lim(x,y)→(2,−1)(4x−3y)=lim(x,y)→(2,−1)4x−lim(x, y)→(2,−1)3y=4(lim(x,y)→(2,−1)x)−3(lim(x,y)→(2,−1)y)=4(2 )−3(−1)=11.


          Поскольку предел знаменателя не равен нулю, применяется закон частного. Теперь вычислим предел числителя, используя разностный закон, закон констант-множителей и закон тождества:

          lim(x,y)→(2,−1)(2x+3y)=lim(x,y)→(2 ,−1)2x+lim(x,y)→(2,−1)3y=2(lim(x,y)→(2,−1)x)+3(lim(x,y)→(2 ,−1)y)=2(2)+3(−1)=1.lim(x,y)→(2,−1)(2x+3y)=lim(x,y)→(2,− 1)2x+lim(x,y)→(2,−1)3y=2(lim(x,y)→(2,−1)x)+3(lim(x,y)→(2,− 1)у)=2(2)+3(-1)=1.


          Таким образом, согласно факторному закону имеем

          lim(x,y)→(2,−1)2x+3y4x−3y=lim(x,y)→(2,−1)(2x+3y)lim (x,y)→(2,−1)(4x−3y)=111. lim(x,y)→(2,−1)2x+3y4x−3y=lim(x,y)→(2,− 1)(2x+3y)lim(x,y)→(2,−1)(4x−3y)=111.

        Контрольно-пропускной пункт 4.6

        Оценить следующий предел:

        lim(x,y)→(5,−2)x2−yy2+x−13.lim(x,y)→(5,−2)x2−yy2+x−13 .

        Поскольку мы берем предел функции двух переменных, точка (a,b)(a,b) находится в ℝ2,ℝ2, и к этой точке можно подойти с бесконечного числа направлений. Иногда при расчете предела ответ меняется в зависимости от выбранного пути к (a,b).(a,b). Если это так, то предела не существует. Другими словами, предел должен быть уникальным, независимо от выбранного пути.

        Пример 4.9

        Несуществующие пределы

        Показать, что не существует ни одного из следующих пределов:

        1. lim(x,y)→(0,0)2xy3x2+y2lim(x,y)→(0,0)2xy3x2+y2
        2. lim(x,y)→(0,0)4xy2x2+3y4lim(x,y)→(0,0)4xy2x2+3y4
        Решение
        1. Область определения функции f(x,y)=2xy3x2+y2f(x,y)=2xy3x2+y2 состоит из всех точек xy-planexy-plane, кроме точки (0,0)(0,0 ) (рис. 4.16). Чтобы показать, что предела не существует, когда (x,y)(x,y) стремится к (0,0),(0,0), заметим, что невозможно удовлетворить определению предела функции двух переменных из-за того, что функция принимает разные значения на разных линиях, проходящих через точку (0,0).(0,0). Сначала рассмотрим линию y=0y=0 в xy-plane.xy-plane. Подстановка y=0y=0 в f(x,y)f(x,y) дает

          f(x,0)=2x(0)3×2+02=0f(x,0)=2x(0)3×2+02=0


          для любого значения x.x. Поэтому значение ff остается постоянным для любой точки на оси x, оси x, и когда yy приближается к нулю, функция остается фиксированной на нуле.
          Далее рассмотрим линию y=x.y=x. Подстановка y=xy=x в f(x,y)f(x,y) дает

          f(x,x)=2x(x)3×2+x2=2x24x2=12.f(x,x)=2x(x )3×2+x2=2x24x2=12.


          Это справедливо для любой точки на прямой y=x.y=x. Если мы позволим xx приблизиться к нулю, оставаясь на этой линии, значение функции останется фиксированным на уровне 12,12, независимо от того, насколько мал xx.
          Выберите для εε значение меньше 1/21/2, скажем, 1/4,1/4. Тогда, независимо от того, насколько маленький диск δδ мы нарисуем вокруг (0,0),(0,0), значения f(x,y)f(x,y) для точек внутри этого диска δδ будут включать как 00, так и 12.12. Следовательно, определение предела в точке никогда не выполняется, и предел не существует.

          Рисунок 4.16 График функции f(x,y)=(2xy)/(3×2+y2).f(x,y)=(2xy)/(3×2+y2). По линии y=0,y=0 функция равна нулю; вдоль линии y=x,y=x функция равна 12.12.


          Подобно а., мы можем приблизиться к началу координат по любой прямой, проходящей через начало координат. Если мы попробуем использовать ось xx (т. е. y=0), y=0), то функция останется зафиксированной на нуле. То же самое верно и для оси y.y-axis. Допустим, мы подходим к началу координат по прямой линии наклона к.к. Уравнение этой линии y=kx.y=kx. Тогда предел становится

          lim(x,y)→(0,0)4xy2x2+3y4=lim(x,y)→(0,0)4x(kx)2×2+3(kx)4=lim(x,y )→(0,0)4k2x3x2+3k4x4=lim(x,y)→(0,0)4k2x1+3k4x2=lim(x,y)→(0,0)(4k2x)lim(x,y)→( 0,0)(1+3k4x2)=0lim(x,y)→(0,0)4xy2x2+3y4=lim(x,y)→(0,0)4x(kx)2×2+3(kx)4= lim(x,y)→(0,0)4k2x3x2+3k4x4=lim(x,y)→(0,0)4k2x1+3k4x2=lim(x,y)→(0,0)(4k2x)lim(x ,y)→(0,0)(1+3k4x2)=0


          независимо от значения k. k. Казалось бы, предел равен нулю. Что если вместо этого мы выберем кривую, проходящую через начало координат? Например, мы можем рассмотреть параболу, заданную уравнением x=y2.x=y2. Замена y2y2 вместо xx в f(x,y)f(x,y) дает

          lim(x,y)→(0,0)4xy2x2+3y4=lim(x,y)→(0,0)4 (y2)y2(y2)2+3y4=lim(x,y)→(0,0)4y4y4+3y4=lim(x,y)→(0,0)1=1.lim(x,y)→ (0,0)4xy2x2+3y4=lim(x,y)→(0,0)4(y2)y2(y2)2+3y4=lim(x,y)→(0,0)4y4y4+3y4=lim (х,у)→(0,0)1=1.


          По той же логике, что и в п., невозможно найти круг δδ вокруг начала координат, удовлетворяющий определению предела для любого значения ε<1.ε<1. Следовательно, lim(x,y)→(0,0)4xy2x2+3y4lim(x,y)→(0,0)4xy2x2+3y4 не существует.

        Контрольно-пропускной пункт 4.7

        Показать, что

        lim(x,y)→(2,1)(x−2)(y−1)(x−2)2+(y−1)2lim(x,y)→(2, 1)(x−2)(y−1)(x−2)2+(y−1)2

        не существует.

        Внутренние точки и граничные точки

        Чтобы изучить непрерывность и дифференцируемость функции двух или более переменных, нам сначала нужно выучить новую терминологию.

        Определение

        Пусть S будет подмножеством ℝ2ℝ2 (рис. 4.17).

        1. Точка P0P0 называется внутренней точкой SS, если существует диск δδ с центром вокруг P0P0, полностью содержащийся в S.S.
        2. Точка P0P0 называется граничной точкой SS, если каждый круг δδ с центром вокруг точки P0P0 содержит точки как внутри, так и вне SS

        Рисунок 4.17 В показанном множестве SS (−1,1)(−1,1) — внутренняя точка, а (2,3)(2,3) — граничная точка.

        Определение

        Пусть S будет подмножеством ℝ2ℝ2 (рис. 4.17).

        1. SS называется открытым множеством, если каждая точка SS является внутренней точкой.
        2. СС называется замкнутым множеством, если оно содержит все свои граничные точки.

        Примером открытого множества является δδ-диск. Если мы включим границу диска, то он станет замкнутым множеством. Множество, которое содержит некоторые, но не все, его граничные точки, не является ни открытым, ни закрытым. Например, если мы включаем половину границы δδ-диска, но не другую половину, то множество не является ни открытым, ни закрытым.

        Определение

        Пусть S будет подмножеством ℝ2ℝ2 (рис. 4.17).

        1. Открытое множество SS является связным множеством, если оно не может быть представлено в виде объединения двух или более непересекающихся непустых открытых подмножеств.
        2. Набор SS является регионом, если он открыт, связан и непуст.

        Определение предела функции двух переменных требует, чтобы диск δδ содержался внутри области определения функции. Однако, если мы хотим найти предел функции в граничной точке области, δdiskδdisk не содержится внутри области. По определению некоторые точки δdiskδdisk находятся внутри области, а некоторые — снаружи. Поэтому нам нужно рассматривать только точки, находящиеся как внутри δδ-диска, так и внутри области определения функции. Это приводит к определению предела функции в граничной точке.

        Определение

        Пусть ff — функция двух переменных, xx и y, y, и пусть (a,b)(a,b) находится на границе области определения f.f. Тогда предел f(x,y)f(x,y) при приближении (x,y)(x,y) к (a,b)(a,b) есть L,L, записанный как

        lim(x ,y)→(a,b)f(x,y)=L,lim(x,y)→(a,b)f(x,y)=L,

        , если для любого ε>0,ε> 0 существует число δ>0δ>0 такое, что для любой точки (x,y)(x,y) внутри области определения ff и на достаточно малом положительном расстоянии δδ от (a,b),(a,b ), значение f(x,y)f(x,y) удалено от LL не более чем на εε (рис. 4.15). Используя символы, мы можем написать: Для любого ε>0,ε>0 существует число δ>0δ>0 такое, что

        |f(x,y)−L|<εwhenever0<(x−a)2+(y−b)2<δ.|f(x,y)−L|<εwhenever0<(x−a)2 +(y−b)2<δ.

        Пример 4.10

        Предел функции в граничной точке

        Доказать lim(x,y)→(4,3)25−x2−y2=0.lim(x,y)→(4,3)25−x2−y2 =0.

        Решение

        Область определения функции f(x,y)=25−x2−y2f(x,y)=25−x2−y2 равна {(x,y)∈ℝ2|x2+y2≤25},{(x, y)∈ℝ2|x2+y2≤25}, который представляет собой окружность радиусом 55 с центром в начале координат, а также ее внутреннюю часть, как показано на следующем графике.

        Рисунок 4.18 Область определения функции f(x,y)=25−x2−y2.f(x,y)=25−x2−y2.

        Мы можем использовать предельные законы, которые применяются к границам областей, а также к внутренним точкам:

        lim(x,y)→(4,3)25−x2−y2=lim(x,y) →(4,3)(25−x2−y2)=lim(x,y)→(4,3)25−lim(x,y)→(4,3)x2−lim(x,y)→( 4,3)y2=25−42−32=0.lim(x,y)→(4,3)25−x2−y2=lim(x,y)→(4,3)(25−x2−y2 )=lim(x,y)→(4,3)25−lim(x,y)→(4,3)x2−lim(x,y)→(4,3)y2=25−42−32= 0.

        См. следующий график.

        Рисунок 4.19График функции f(x,y)=25−x2−y2.f(x,y)=25−x2−y2.

        Контрольно-пропускной пункт 4,8

        Оцените следующий предел:

        lim(x,y)→(5,−2)29−x2−y2.lim(x,y)→(5,−2)29−x2−y2.

        Непрерывность функций двух переменных

        В Непрерывности мы определили непрерывность функции одной переменной и увидели, как она зависит от предела функции одной переменной. В частности, для непрерывности f(x)f(x) в точке x=a:x=a необходимы три условия:

        1. f(a)f(a) существует.
        2. limx→af(x)limx→af(x) существует.
        3. limx→af(x)=f(a).limx→af(x)=f(a).

        Эти три условия необходимы и для непрерывности функции двух переменных.

        Определение

        Функция f(x,y)f(x,y) непрерывна в точке (a,b)(a,b) в своей области определения, если выполняются следующие условия:

        1. f(a,b) f(a,b) существует.
        2. lim(x,y)→(a,b)f(x,y)lim(x,y)→(a,b)f(x,y) существует.
        3. lim(x,y)→(a,b)f(x,y)=f(a,b).lim(x,y)→(a,b)f(x,y)=f(a, б).

        Пример 4.11

        Демонстрация непрерывности функции двух переменных

        Показать, что функция f(x,y)=3x+2yx+y+1f(x,y)=3x+2yx+y+1 непрерывна в точке (5, −3).(5, −3).

        Решение

        Согласно определению непрерывности должны быть выполнены три условия. В этом примере a=5a=5 и b=-3.b=-3.

        1. f(a,b)f(a,b) существует. Это верно, потому что область определения функции ff состоит из тех упорядоченных пар, для которых знаменатель отличен от нуля (т. е. x+y+1≠0).x+y+1≠0). Точка (5,−3)(5,−3) удовлетворяет этому условию. Кроме того,

          f(a,b)=f(5,−3)=3(5)+2(−3)5+(−3)+1=15−62+1=3.f(a,b)= f(5,−3)=3(5)+2(−3)5+(−3)+1=15−62+1=3.

        2. lim(x,y)→(a,b)f(x,y)lim(x,y)→(a,b)f(x,y) существует. Это также верно:

          lim(x,y)→(a,b)f(x,y)=lim(x,y)→(5,−3)3x+2yx+y+1=lim(x, y)→(5,−3)(3x+2y)lim(x,y)→(5,−3)(x+y+1)=15−65−3+1=3.lim(x,y )→(a,b)f(x,y)=lim(x,y)→(5,−3)3x+2yx+y+1=lim(x,y)→(5,−3)(3x +2y)lim(x,y)→(5,−3)(x+y+1)=15−65−3+1=3.

        3. lim(x,y)→(a,b)f(x,y)=f(a,b).lim(x,y)→(a,b)f(x,y)=f(a, б). Это верно, потому что мы только что показали, что обе части этого уравнения равны трем.

        Контрольно-пропускной пункт 4.9

        Показать, что функция f(x,y)=26−2×2−y2f(x,y)=26−2×2−y2 непрерывна в точке (2,−3).(2,−3).

        Непрерывность функции любого числа переменных также может быть определена в терминах дельта и эпсилон. Функция двух переменных непрерывна в точке (x0,y0)(x0,y0) своей области определения, если для каждого ε>0ε>0 существует δ>0δ>0 такое, что всякий раз, когда (x−x0)2+ (y−y0)2<δ(x−x0)2+(y−y0)2<δ верно, |f(x,y)−f(a,b)|<ε. |f(x, y)−f(a,b)|<ε. Это определение можно комбинировать с формальным определением (то есть эпсилон-дельта определение ) непрерывности функции одной переменной для доказательства следующих теорем:

        Теорема 4.2

        Сумма непрерывных функций непрерывна

        Если f(x,y)f(x,y) непрерывна в точках (x0,y0),(x0,y0) и g(x,y)g(x,y) непрерывна в точках (x0,y0 ),(x0,y0), то f(x,y)+g(x,y)f(x,y)+g(x,y) непрерывна в (x0,y0).(x0,y0).

        Теорема 4.3

        Произведение непрерывных функций непрерывно

        Если g(x)g(x) непрерывна в точке x0x0 и h(y)h(y) непрерывна в точках y0,y0, то f(x,y)=g(x)h(y)f(x, y)=g(x)h(y) непрерывна в точке (x0,y0).(x0,y0).

        Теорема 4.4

        Композиция непрерывных функций непрерывна

        Пусть gg — функция двух переменных из области определения D⊆ℝ2D⊆ℝ2 в диапазон R⊆ℝ.R⊆ℝ. Предположим, что gg непрерывна в некоторой точке (x0,y0)∈D(x0,y0)∈D, и определим z0=g(x0,y0). z0=g(x0,y0). Пусть ff — функция, отображающая ℝℝ в ℝℝ, такая, что z0z0 находится в области определения f.f. Наконец, предположим, что ff непрерывна в точке z0.z0. Тогда f∘gf∘g непрерывна в точке (x0,y0)(x0,y0), как показано на следующем рисунке.

        Рисунок 4.20 Композиция двух непрерывных функций непрерывна.

        Теперь воспользуемся предыдущими теоремами, чтобы показать непрерывность функций в следующих примерах.

        Пример 4.12

        Дополнительные примеры непрерывности функции двух переменных

        Показать, что функции f(x,y)=4x3y2f(x,y)=4x3y2 и g(x,y)=cos(4x3y2)g(x,y )=cos(4x3y2) всюду непрерывны.

        Решение

        Многочлены g(x)=4x3g(x)=4×3 и h(y)=y2h(y)=y2 непрерывны при каждом вещественном числе, и поэтому по теореме о произведении непрерывных функций f(x,y)= 4x3y2f(x,y)=4x3y2 непрерывно в каждой точке (x,y)(x,y) плоскости xy.xy-plane. Поскольку f(x,y)=4x3y2f(x,y)=4x3y2 непрерывна в каждой точке (x,y)(x,y) плоскости xy-xy и g(x)=cosxg(x)=cosx непрерывна в каждом вещественном числе x,x, непрерывность композиции функций говорит нам, что g(x,y)=cos(4x3y2)g(x,y)=cos(4x3y2) непрерывна в каждой точке (x, y)(x,y) в xy-plane. xy-plane.

        Контрольно-пропускной пункт 4.10

        Показать, что функции f(x,y)=2x2y3+3f(x,y)=2x2y3+3 и g(x,y)=(2x2y3+3)4g(x,y)=(2x2y3+3) 4 непрерывны всюду.

        Функции трех и более переменных

        Предел функции трех или более переменных легко возникает в приложениях. Например, предположим, что у нас есть функция f(x,y,z)f(x,y,z), которая дает температуру в физическом местоположении (x,y,z)(x,y,z) в трех измерениях. Или, возможно, функция g(x,y,z,t)g(x,y,z,t) может показывать давление воздуха в точке (x,y,z)(x,y,z) в момент времени t.t. Как мы можем взять предел в точке в ℝ3?ℝ3? Что значит быть непрерывным в точке в четырех измерениях?

        Ответы на эти вопросы основаны на расширении концепции δδ-диска более чем на два измерения. Тогда представления о пределе функции трех и более переменных и непрерывности функции трех и более переменных очень похожи на определения, данные ранее для функции двух переменных.

        Определение

        Пусть (x0,y0,z0)(x0,y0,z0) точка в ℝ3. ℝ3. Тогда шар δδ в трех измерениях состоит из всех точек из ℝ3ℝ3, лежащих на расстоянии меньше δδ от (x0,y0,z0)(x0,y0,z0), т. е.

        {(x,y,z)∈ℝ3|(x−x0)2+(y−y0)2+(z−z0)2<δ}.{(x,y,z)∈ℝ3|(x −x0)2+(y−y0)2+(z−z0)2<δ}.

        Чтобы определить шар δδ в более высоких измерениях, добавьте дополнительные термины под радикалом, чтобы они соответствовали каждому дополнительному измерению. Например, для точки P=(w0,x0,y0,z0)P=(w0,x0,y0,z0) в ℝ4, ℝ4 шар δδ вокруг PP можно описать как

        {(w,x, y,z)∈ℝ4|(w−w0)2+(x−x0)2+(y−y0)2+(z−z0)2<δ}.{(w,x,y,z)∈ℝ4 |(w−w0)2+(x−x0)2+(y−y0)2+(z−z0)2<δ}.

        Чтобы показать, что предел функции трех переменных существует в точке (x0,y0,z0),(x0,y0,z0), достаточно показать, что для любой точки δδ-шара с центром в (x0,y0 ,z0),(x0,y0,z0), значение функции в этой точке сколь угодно близко к фиксированному значению (предельному значению). Все предельные законы для функций двух переменных справедливы и для функций более чем двух переменных.

        Пример 4.

        13
        Нахождение предела функции трех переменных

        Найти lim(x,y,z)→(4,1,−3)x2y−3z2x+5y−z.lim(x,y,z)→(4 ,1,−3)x2y−3z2x+5y−z.

        Решение

        Прежде чем мы сможем применить частное, нам нужно убедиться, что предел знаменателя отличен от нуля. Используя разностный закон, закон тождества и постоянный закон,

        lim(x,y,z)→(4,1,−3)(2x+5y−z)=2(lim(x,y,z )→(4,1,−3)x)+5(lim(x,y,z)→(4,1,−3)y)−(lim(x,y,z)→(4,1, −3)z)=2(4)+5(1)−(−3)=16.lim(x,y,z)→(4,1,−3)(2x+5y−z)=2( lim(x,y,z)→(4,1,−3)x)+5(lim(x,y,z)→(4,1,−3)y)−(lim(x,y,z )→(4,1,−3)z)=2(4)+5(1)−(−3)=16.

        Так как это не ноль, мы затем находим предел числителя. Используя закон произведения, разностный закон, постоянный кратный закон и закон тождества,

        lim(x,y,z)→(4,1,−3)(x2y−3z)=(lim(x,y,z) →(4,1,−3)x)2(lim(x,y,z)→(4,1,−3)y)−3lim(x,y,z)→(4,1,−3) z=(42)(1)−3(−3)=16+9=25.lim(x,y,z)→(4,1,−3)(x2y−3z)=(lim(x,y ,z)→(4,1,−3)x)2(lim(x,y,z)→(4,1,−3)y)−3lim(x,y,z)→(4,1, −3)z=(42)(1)−3(−3)=16+9=25.

        Наконец, применяя частное:

        lim(x,y,z)→(4,1,−3)x2y−3z2x+5y−z=lim(x,y,z)→(4,1, −3)(x2y−3z)lim(x,y,z)→(4,1,−3)(2x+5y−z)=2516.lim(x,y,z)→(4,1,− 3)x2y−3z2x+5y−z=lim(x,y,z)→(4,1,−3)(x2y−3z)lim(x,y,z)→(4,1,−3)( 2x+5y−z)=2516.

        Контрольно-пропускной пункт 4.11

        Найти lim(x,y,z)→(4,−1,3)13−x2−2y2+z2.lim(x,y,z)→(4,−1,3)13−x2−2y2 +z2.

        Раздел 4.2 Упражнения

        Для следующих упражнений найдите предел функции.

        60.

        lim(x,y)→(1,2)xlim(x,y)→(1,2)x

        61.

        lim(x,y)→(1,2)5x2yx2+y2lim(x,y)→(1,2)5x2yx2+y2

        62.

        Показать, что предел lim(x,y)→(0,0)5x2yx2+y2lim(x,y)→(0,0)5x2yx2+y2 существует и одинаков вдоль путей: y-осьy-ось и ось x, ось x и вдоль y=x.y=x.

        Для следующих упражнений оцените пределы при указанных значениях xandy.xandy. Если предела не существует, укажите это и объясните, почему предела не существует.

        63.

        lim(x,y)→(0,0)4×2+10y2+44×2−10y2+6lim(x,y)→(0,0)4×2+10y2+44×2−10y2+6

        64.

        lim(x,y)→(11,13)1xylim(x,y)→(11,13)1xy

        65.

        lim(x,y)→(0,1)y2sinxxlim(x,y)→(0,1)y2sinxx

        66.

        lim(x,y)→(0,0)sin(x8+y7x−y+10)lim(x,y)→(0,0)sin(x8+y7x−y+10)

        67.

        lim(x,y)→(π/4,1)ytanxy+1lim(x,y)→(π/4,1)ytanxy+1

        68.

        lim(x,y)→(0,π/4)secx+23x−tanylim(x,y)→(0,π/4)secx+23x−tany

        69.

        lim(x,y)→(2,5)(1x−5y)lim(x,y)→(2,5)(1x−5y)

        70.

        lim(x,y)→(4,4)xlnylim(x,y)→(4,4)xlny

        71.

        lim(x,y)→(4,4)e−x2−y2lim(x,y)→(4,4)e−x2−y2

        72.

        lim(x,y)→(0,0)9−x2−y2lim(x,y)→(0,0)9−x2−y2

        73.

        lim(x,y)→(1,2)(x2y3−x3y2+3x+2y)lim(x,y)→(1,2)(x2y3−x3y2+3x+2y)

        74.

        lim(x,y)→(π,π)xsin(x+y4)lim(x,y)→(π,π)xsin(x+y4)

        75.

        lim(x,y)→(0,0)xy+1×2+y2+1lim(x,y)→(0,0)xy+1×2+y2+1

        76.

        lim(x,y)→(0,0)x2+y2x2+y2+1−1lim(x,y)→(0,0)x2+y2x2+y2+1−1

        77.

        lim(x,y)→(0,0)ln(x2+y2)lim(x,y)→(0,0)ln(x2+y2)

        Для следующих упражнений завершите утверждение.

        78.

        Точка (x0,y0)(x0,y0) в плоской области RR является внутренней точкой RR, если _________________.

        79.

        Точка (x0,y0)(x0,y0) в плоской области RR называется граничной точкой RR, если ___________.

        В следующих упражнениях используйте алгебраические методы для оценки предела.

        80.

        lim(x,y)→(2,1)x−y−1x−y−1lim(x,y)→(2,1)x−y−1x−y−1

        81.

        lim(x,y)→(0,0)x4−4y4x2+2y2lim(x,y)→(0,0)x4−4y4x2+2y2

        82.

        lim(x,y)→(0,0)x3−y3x−ylim(x,y)→(0,0)x3−y3x−y

        83.

        lim(x,y)→(0,0)x2−xyx−ylim(x,y)→(0,0)x2−xyx−y

        В следующих упражнениях оцените пределы функций трех переменных.

        84.

        lim(x,y,z)→(1,2,3)xz2−y2zxyz−1lim(x,y,z)→(1,2,3)xz2−y2zxyz−1

        85.

        lim(x,y,z)→(0,0,0)x2−y2−z2x2+y2−z2lim(x,y,z)→(0,0,0)x2−y2−z2x2+y2−z2

        Для следующих упражнений оцените предел функции, определив значение, к которому функция приближается по указанным путям. Если предела нет, объясните, почему.

        86.

        lim(x,y)→(0,0)xy+y3x2+y2lim(x,y)→(0,0)xy+y3x2+y2

        1. Вдоль оси xx-axis(y=0)(y =0)
        2. Вдоль оси Yy-ось (x=0)(x=0)
        3. По пути y=2xy=2x

        87.

        Вычислить lim(x,y)→(0,0)xy+y3x2+y2lim(x,y)→(0,0)xy+y3x2+y2, используя результаты предыдущей задачи.

        88.

        lim(x,y)→(0,0)x2yx4+y2lim(x,y)→(0,0)x2yx4+y2

        1. Вдоль оси x (y=0)(y=0)
        2. Вдоль оси y (x=0)(x=0)
        3. По пути y=x2y=x2

        89.

        Вычислить lim(x,y)→(0,0)x2yx4+y2lim(x,y)→(0,0)x2yx4+y2, используя результаты предыдущей задачи.

        Обсудите непрерывность следующих функций. Найдите наибольшую область в xy-planexy-плоскости, в которой следующие функции непрерывны.

        90.

        f(x,y)=sin(xy)f(x,y)=sin(xy)

        91.

        f(x,y)=ln(x+y)f(x,y)=ln(x+y)

        92.

        f(x,y)=e3xyf(x,y)=e3xy

        93.

        f(x,y)=1xyf(x,y)=1xy

        Для следующих упражнений определите область, в которой функция непрерывна. Поясните свой ответ.

        94.

        f(x,y)=x2yx2+y2f(x,y)=x2yx2+y2

        95.

        f(x,y)={x2yx2+y2if(x,y)≠(0,0)0if(x,y)=(0,0)}f(x,y)={x2yx2+y2if(x,y) )≠(0,0)0if(x,y)=(0,0)}

        ( Подсказка : Покажите, что функция приближается к разным значениям двумя разными путями.)

        96.

        f(x,y)=sin(x2+y2)x2+y2f(x,y)=sin(x2+y2)x2+y2

        97.

        Определить, является ли g(x,y)=x2−y2x2+y2g(x,y)=x2−y2x2+y2 непрерывным в точке (0,0).(0,0).

        98.

        Создайте график с помощью графического программного обеспечения, чтобы определить, где предел не существует. Найдите, где в координатной плоскости функция f(x,y)=1×2−yf(x,y)=1×2−y непрерывна.

        99.

        Определите область xy-planexy-plane, в которой функция g(x,y)=arctan(xy2x+y)g(x,y)=arctan(xy2x+y) непрерывна. Используйте технологии, чтобы подтвердить свой вывод.

        100.

        Определите область xy-плоскости xy-плоскости, в которой f(x,y)=ln(x2+y2−1)f(x,y)=ln(x2+y2−1) непрерывна. Используйте технологии, чтобы подтвердить свой вывод. ( Совет : Тщательно выбирайте диапазон значений для xandyxandy!)

        101.

        В каких точках пространства g(x,y,z)=x2+y2−2z2g(x,y,z)=x2+y2−2z2 непрерывен?

        102.

        В каких точках пространства g(x,y,z)=1×2+z2−1g(x,y,z)=1×2+z2−1 непрерывен?

        103.

        Показать, что lim(x,y)→(0,0)1×2+y2lim(x,y)→(0,0)1×2+y2 не существует в точке (0,0)(0,0), построив график график функции.

        104.

        [T] Оцените lim(x,y)→(0,0)−xy2x2+y4lim(x,y)→(0,0)−xy2x2+y4, построив график функции с помощью CAS. Определить аналитически предел по пути x=y2. x=y2.

        105.

        [Т]

        1. Используйте CAS, чтобы нарисовать контурную карту z=9-x2-y2.z=9-x2-y2.
        2. Как называется геометрическая форма кривых уровня?
        3. Приведите общее уравнение кривых уровня.
        4. Каково максимальное значение z?z?
        5. Какова область определения функции?
        6. Каков диапазон функции?

        106.

        Верно или неверно : Если мы оцениваем lim(x,y)→(0,0)f(x)lim(x,y)→(0,0)f(x) по нескольким путям и каждый раз, когда предел равен 1,1, мы можем заключить, что lim(x,y)→(0,0)f(x)=1.lim(x,y)→(0,0)f(x)=1.

        107.

        Используйте полярные координаты, чтобы найти lim(x,y)→(0,0)sinx2+y2x2+y2.lim(x,y)→(0,0)sinx2+y2x2+y2. Вы также можете найти предел, используя правило Лопиталя.

        108.

        Используйте полярные координаты, чтобы найти lim(x,y)→(0,0)cos(x2+y2). lim(x,y)→(0,0)cos(x2+y2).

        109.

        Обсудите непрерывность f(g(x,y))f(g(x,y)) где f(t)=1/tf(t)=1/t и g(x,y)=2x− 5y.g(x,y)=2x−5y.

        110.

        Учитывая f(x,y)=x2−4y,f(x,y)=x2−4y, найти limh→0f(x+h,y)−f(x,y)h.limh→0f(x +h,y)−f(x,y)h.

        111.

        Учитывая f(x,y)=x2−4y,f(x,y)=x2−4y, найти limh→0f(1+h,y)−f(1,y)h.limh→0f(1 +h,y)−f(1,y)h.

        14.2 Пределы и непрерывность

        Чтобы разработать исчисление для функций одной переменной, нам нужно было сделать смысл понятия предела, который нам нужно было понять непрерывные функции и определить производную. Ограничения, связанные функции двух переменных могут быть значительно сложнее иметь дело с; к счастью, большинство функций, с которыми мы сталкиваемся, довольно просты чтобы понять.

        Потенциальные трудности во многом связаны с тем, что есть много способов «приблизиться» к точке на плоскости $x$-$y$. Если мы хотим скажем, что $\ds\lim_{(x,y)\to(a,b)}f(x,y)=L$, нам нужно захватить идея о том, что когда $(x,y)$ приближается к $(a,b)$, тогда $f(x,y)$ приближается к $L$. Для функций одной переменной $f(x)$ есть только два способа что $x$ может приближаться к $a$: слева или справа. Но есть бесконечное количество способов приблизиться к $(a,b)$: по любому из бесконечное число линий, или бесконечное число парабол, или бесконечное количество синусоид и так далее. Мы могли бы надеяться, что это на самом деле не так уж плохо — предположим, например, что вдоль всех возможных через $(a,b)$ значение $f(x,y)$ приближается к $L$; конечно это означает, что «$f(x,y)$ приближается к $L$, когда $(x,y)$ приближается к $(а,б)$». К сожалению нет. 92$. Приближаясь к начало пути по прямой, переходим через гребень и спускаемся вниз к 0, но приближаясь по гребню высота постоянна $1/2$. Таким образом, в $(0,0)$ нет предела. $\квадрат$

        К счастью, мы можем определить понятие предела без необходимости указать, как приближаться к конкретной точке — действительно, в определение 2. n$, где $a$ — действительное число, а $m$ и $n$ — целые неотрицательные числа. Рациональное функция является частным многочленов. 92}$? Объяснять.

        Узнать о пределе функции двух переменных

        Предел функции двух переменных Определение

        Предел функции — это точка, в которой функция может быть проверена на непрерывность. Говорят, что функция приближается к своему пределу при условиях, применяемых к ее области определения.

        Обзор предела функции двух переменных

        Забег на 100 метров обычно является частью спортивного дня любой школы. Джон, Майк и Ной участвуют в этой гонке, организованной их школой в ежегодный день спорта. Ной выигрывает гонку с очень небольшим отрывом от Майка. Теперь, исходя из мгновенной скорости Майка и Ноя, этот небольшой запас во времени можно легко определить с помощью пределов. Здесь расстояние между участниками и финишной чертой приближается к нулю, поскольку их скорость приближается к максимально возможному потенциалу. Используя эту информацию и применяя ограничения как на расстояние, так и на скорость, можно легко получить время, за которое Майк и Ной пересекают финишную черту, и, следовательно, также разницу, которая объявляет Ноя победителем. Это не кажется чем-то большим, но подобные примеры демонстрируют использование ограничений функции, одной или двух переменных, в самых незначительных действиях в нашей повседневной жизни.

        Предел функции двух переменных лежит в основе большей части расширенного исчисления, а также широко используется в физике, химии и большинстве инженерных дисциплин. Чтобы разработать двигатель автомобиля, механик не создает весь узел за один раз. Детали, из которых состоит двигатель, создаются и тестируются независимо друг от друга с использованием небольших приближений, а затем успешно монтируются в единое целое. Эти приближения используют понятия пределов.

        Есть вопрос по этой теме?

        Что вы изучите:

        • Предел функции двух переменных Определение
        • Обзор предела функции двух переменных
        • Математическое определение и представление предела функции двух переменных
        • Математическое моделирование для ограничения Функция двух переменных:
        • Свойства пределов функции двух переменных

        Математическое определение и представление предела функции двух переменных

        92}} < \delta }0<(x−a)2+(y−b)2​<δ

        Пределы функций двух переменных определяются как вправо) _ {\ влево ( {x, y} \ right) \ to \ left ( {a, b} \ right)}} = \ left ( {x — b} \ right) \ left ( {y — a} \right)f(x,y)(x,y)→(a,b)​=(x−b)(y−a), где x и y — две переменные или параметры функции f(x, y)f\left( {x,y} \right)f(x,y), в котором пределы накладываются как x→ax \to ax→a и y→by \to by→b. Когда значение x и y приближается к a и b, то значением функции является результирующее значение функции в точной точке a и b при условии, что переменные a и b независимы, т. е. f(x ,y)f\left( {x,y} \right)f(x,y) будет равно 92}f(a,b)=(a−b)(b−a)=−(a−b)2.

        Математическое моделирование для ограничения функции двух переменных:

        Мальчик выполнял свой проект по химии, в котором он должен был определить конечную концентрацию химического вещества X, когда химические вещества Y и Z вступали в реакцию, образуя новый химический состав X. Здесь , концентрации химических веществ были приняты в качестве двух независимых переменных, поскольку концентрация химических веществ изменяется в каждый момент времени от нового химического вещества X , которое рассматривается как функция Y и Z. Здесь, поскольку химические вещества Y и Z реагируют со временем, концентрация химического вещества X увеличивается со временем. { \pm \infty }}} \right)(±∞±∞;∞×∞;0±∞) тогда это будет значение функции, и этот метод известен как метод прямой подстановки. Пределы на функцию двух переменных могут быть наложены бесконечным числом способов, как и в случае окружности. Здесь мы видим, что функцию радиуса можно определить от начала координат до конечных точек на окружности окружности. По мере уменьшения расстояния между центром и конечными точками радиус окружности также будет последовательно уменьшаться, и анализ будет выполняться соответствующим образом.

        Свойства пределов функции двух переменных

        Для двух функций f(x,y)f\left( {x,y} \right)f(x,y) и g(x,y)g\ left( {x,y} \right)g(x,y) , если гипотеза lim⁡(x,y)→(a,b)f(x,y)=M\mathop {\lim }\limits_{ \left( {x,y} \right) \to \left( {a,b} \right)} f\left( {x,y} \right) = M(x,y)→(a,b) lim​f(x,y)=M и lim⁡(x,y)→(a,b)g(x,y)=N\mathop {\lim }\limits_{\left({x,y} \ вправо) \к\влево( {а,b} \вправо)} g\влево( {x,y} \вправо) = N(x,y)→(a,b)lim​g(x,y)= N, то будет определен следующий вывод (свойства): • Линейность (Масштабирование): Когда постоянный член умножается на функцию, на которую должны быть наложены ограничения, тогда этот общий член может быть взят из пределов, поскольку член не зависит от переменных, зарегистрированных в функции. Математически

        lim⁡(x,y)→(a,b)cf(x,y)=c[lim⁡(x,y)→(a,b)f(x,y)]\mathop {\lim} \limits_{\left({x,y} \right) \to \left({a,b} \right)} cf\left({x,y} \right) = c\left[ {\mathop {\ lim }\limits_{\left( {x,y} \right) \to \left({a,b} \right)} f\left( {x,y} \right)} \right](x,y )→(a,b)lim​cf(x,y)=c[(x,y)→(a,b)lim​f(x,y)] и lim⁡(x,y)→(a, б) cg(x,y)=c[lim⁡(x,y)→(a,b)g(x,y)]\mathop {\lim}\limits_{\left({x,y} \right ) \to \left( {a,b} \right)} cg\left( {x,y} \right) = c\left[ {\mathop {\lim }\limits_{\left({x,y} \right) \to \left( {a,b} \right)} g\left( {x,y} \right)} \right](x,y)→(a,b)lim​cg(x, y)=c[(x,y)→(a,b)lim​g(x,y)]

        • Добавление функций: когда несколько функций складываются вместе таким образом, что пределы всех функций одинаковы с одинаковыми параметрами или переменными, тогда функции могут быть добавлены вместе, а затем, наконец, наложены ограничения. Математически, lim⁡(x,y)→(a,b)(f+g)(x,y)=lim⁡(x,y)→(a,b)f(x,y)+lim⁡(x,y )→(a,b)g(x,y)\mathop {\lim }\limits_{\left({x,y} \right) \to \left({a,b} \right)} \left( {f + g} \right)\left( {x,y} \right) = \mathop {\lim }\limits_{\left({x,y} \right) \to \left({a,b} \right)} f\left( {x,y} \right) + \mathop {\lim }\limits_{\left({x,y} \right) \to \left({a,b} \right) } g\left( {x,y} \right)(x,y)→(a,b)lim​(f+g)(x,y)=(x,y)→(a,b)lim​ f(x,y)+(x,y)→(a,b)lim​g(x,y)

        • Произведение функций: когда несколько функций перемножаются таким образом, что пределы всех функций одинаковы с одинаковыми параметрами или переменными, тогда функции могут быть перемножены вместе, а затем, наконец, наложены ограничения.

      Добавить комментарий

      Ваш адрес email не будет опубликован. Обязательные поля помечены *